Family Medicine EOR

अब Quizwiz के साथ अपने होमवर्क और परीक्षाओं को एस करें!

Question: What are the metabolic abnormalities associated with tumor lysis syndrome?

Answer: Hyperphosphatemia, hypocalcemia, hyperuricemia, and hyperkalemia.

What are some cardiac dysrhythmias that may be seen on electrocardiogram in patients with syncope?

Brugada syndrome, short QT syndrome, long QT syndrome, preexcitation, and heart block.

What medication improves intermittent claudication in patients with peripheral artery disease?

Cilostazol

What form of cellulitis is characterized by a bright red plaque with raised and sharply demarcated borders?

erysipelas

A 21-year-old man presents to his primary care provider with complaints of right ankle pain for 12 hours. He is a competitive swimmer and has a history of tinea pedis. He has a fever of 102°F and a complete blood count significant for leukocytosis with a left shift. He is not on any medications. Which of the following physical examination findings is most consistent with the suspected diagnosis? A. A single, erythematous patch with overlying warmth B. Multiple lesions with three concentric zones of color change C. Purple, pruritic, polygonal, planar, papular plaques D. Raised, purpuric, target-like lesions with two zones of color change

A. A single, erythematous patch with overlying warmth Multiple lesions with three concentric zones of color change (B) is a classic description of erythema multiforme, an acute inflammatory dermatologic condition associated with herpes simplex virus. Lesions typically occur on the extensor surface of the arms and legs before spreading to the trunk. The oral, ocular, or genital mucosa may also be affected. Purple, pruritic, polygonal, planar, papular plaques (C) are associated with lichen planus, an inflammatory dermatologic condition associated with hepatitis C virus. Lesions may be found on the ankles or wrists as well as the oral and genital mucosa. Raised, purpuric, target-like lesions with two zones of color change (D) is consistent with Stevens-Johnson syndrome and toxic epidermal necrolysis. Toxic epidermal necrolysis affects > 30% of the body surface, while Stevens-Johnson syndrome affects < 10%. Mucocutaneous lesions first appear on the face and trunk and spread outward symmetrically. Involvement of the scalp, palms, and soles is rare. These two conditions are most commonly caused by reactions to medications, such as sulfonamides, nonsteroidal anti-inflammatory drugs, allopurinol, and anticonvulsants.

A 12-year-old boy accompanied by his mother presents to a gastroenterologist for a discussion about colonoscopy. His father had > 1,000 polyps on colonoscopy and passed away at age 55 while he was awaiting a proctocolectomy. The gastroenterologist explains to the patient and mother that he will need an annual colonoscopy for the rest of his life because he tested positive for a certain genetic mutation. Which of the following abnormal genes is associated with the patient's diagnosis? A. APC B. EPCAM C. MLH1 D. PMS2

A. APC Lynch syndrome, also known as hereditary nonpolyposis colorectal cancer, is caused by a germline mutation of several DNA mismatch genes or a deletion of EPCAM (B). It is an autosomal dominant disorder and the most common cause of inherited colorectal cancer. It is also associated with gastric, small intestine, endometrial, ovarian, urinary tract, skin, and pancreatic cancer. Lynch syndrome is associated with right-sided, large tumors and rapid development of cancer, while FAP is characterized by slower development to cancer and bowel-constricting tumors. Lynch syndrome has an older age of onset (mean age 44) compared with FAP (mean age 16). Additionally, in Lynch syndrome, polyps are either absent or few in number. MLH1 (C), MSH2, MSH6, and PMS2 (D) are mutated mismatch repair genes associated with Lynch syndrome. The Amsterdam criteria are used to identify patients at increased risk for Lynch syndrome and can be remembered by the 3-2-1 rule of three family members with Lynch syndrome-associated cancers, Lynch syndrome-associated cancers involving at least two generations, and one or more cancers diagnosed before 50 years of age. Management and treatment of Lynch syndrome include annual colonoscopies beginning at 25 years of age, regular endometrial and ovarian screening, surgical resection, and chemotherapy.

A 30-year-old receptionist presents to her primary care provider with complaints of bilateral wrist pain, tingling, and intermittent numbness. She reports her symptoms are worse at night. She also states her symptoms only affect her thumbs, index fingers, middle fingers, and half of her ring finger. Which of the following findings on physical examination is associated with a risk factor for the most likely diagnosis? A. Acanthosis nigricans B. Chvostek sign C. Hypothenar atrophy D. Positive Adson test

A. Acanthosis nigricans Chvostek sign (B) is hyperexcitability upon tapping of the facial nerve seen in hypocalcemia and is not associated with carpal tunnel syndrome. Hypothenar atrophy (C) is related to pathology of the ulnar nerve, which may be seen with cubital tunnel syndrome. Carpal tunnel syndrome is associated with compression of the median nerve and thenar atrophy. A positive Adson test (D) is indicative of thoracic outlet syndrome. Thoracic outlet syndrome is characterized by compression of the subclavian artery by a cervical rib or tightened anterior and middle scalene muscles.

An 84-year-old woman is brought to a primary care provider by her granddaughter with concerns for dementia. The granddaughter states the patient does not seem to recognize family members anymore and is having a hard time remembering how to perform routine daily tasks. Her Montreal Cognitive Assessment score indicates moderate impairment. Which of the following describes the pathology of the most likely diagnosis? A. Accumulation of hyperphosphorylated tau protein B. Accumulation of Negri bodies in the hippocampus C. Increased CAG trinucleotide repeats D. Increased CGG trinucleotide repeats

A. Accumulation of hyperphosphorylated tau protein Accumulation of Negri bodies in the hippocampus (B) is associated with the rabies virus. Increased CAG trinucleotide repeats (C) results in the production of huntingtin proteins that are associated with Huntington disease. Increased CGG trinucleotide repeats (D) results in the loss of FMR1 gene expression and is associated with fragile X syndrome. Alzheimer disease is the most common cause of dementia and is characterized by cognitive decline. Risk factors include family history, genetic mutations affecting amyloid in the brain, the presence of the apolipoprotein E epsilon 4 allele, hypertension, obesity, and diabetes. Definitive diagnosis requires a brain biopsy, which is rarely performed. Histopathologic findings indicative of Alzheimer disease include amyloid plaques and the accumulation of hyperphosphorylated tau protein in neurofibrillary tangles within the brain. Current hypotheses suggest these amyloid plaques and tau proteins are toxic to the neurons in the brain, resulting in progressive cognitive decline. Older patients with a clinical presentation of progressive memory impairment may be screened with the Mini-Mental Status Examination or the Montreal Cognitive Assessment. Participation in social, mental, and physical activities may decrease the risk of developing Alzheimer disease and other forms of dementia. While there is no cure for Alzheimer disease, cholinesterase inhibitors, such as donepezil, galantamine, and rivastigmine, are used to treat symptoms of cognition and global functioning.

A 22-year-old man presents to the emergency department with thoughts of harming himself. He states he was planning on taking a large amount of medication but was stopped by a friend who brought him to the hospital. A thorough physical and medical evaluation are initiated. Which of the following historical findings would put this patient most at risk for suicide? A. Being a gay man B. Being married C. Family connectedness D. Higher education status

A. Being a gay man Single individuals are more at risk for suicide than their married (B) counterparts. Social support and family connectedness (C) are protective factors against suicide. Lower education status is also associated with an increased risk of suicide. Higher education status (D) is not associated with greater risk of suicide.

A 24-year-old woman presents for her routine checkup. She feels well and has no complaints. Her medical history is significant for seizures, situational anxiety, body dysmorphic disorder, tobacco use disorder, and seasonal allergies. She reports no asthma. She is currently taking lamotrigine, sertraline, loratadine, and fluticasone. She is a daily smoker and has a 5 pack-year history. She has tried over-the-counter nicotine replacement therapies but did not find success with gum or lozenges. She mentions she would like to quit smoking. During the appointment, she sets a quit date 3 weeks from now, texts her family and friends asking for support in her decision, discusses challenges she may encounter when abstaining from tobacco use, and discards the pack of cigarettes she has in her purse. She is interested in beginning a prescription medication to assist her with quitting. Which of the following pharmacotherapies for smoking cessation is contraindicated in this patient? A. Bupropion B. Clonidine C. Nicotine inhaler D. Varenicline

A. Bupropion Clonidine (B) is a second-line therapy for smoking cessation. It may be considered in patients who do not find success with first-line therapies. Patients should be reliable with medication use prior to initiating clonidine therapy since abrupt discontinuation of this medication can cause rapid and dangerous elevation of blood pressure. If the patient described in the vignette does not find success with other first-line therapies, this may be an appropriate second-line option. A nicotine inhaler (C) is an available option for this patient. It has the advantage of mimicking the same hand-to-mouth action as smoking. It may be contraindicated in patients with severe restrictive airway disease. It is appropriate for this patient (although she has seasonal allergies, she does not have asthma). Varenicline (D) is an available treatment option for this patient. Varenicline has the highest cessation rate (roughly 30%). It should be used with caution in patients with manic depression, schizophrenia, or alcohol use disorder.

A 65-year-old man with a history of congestive heart failure and hyperlipidemia presents to the office for management of symptoms related to congestive heart failure. He reports dyspnea on exertion. His vital signs are within normal limits. Electrocardiogram reveals normal sinus rhythm and left ventricular hypertrophy. A recent echocardiogram reveals an ejection fraction of 40%. His current daily medications include atorvastatin, furosemide, and lisinopril. Which of the following medications would be most appropriate to add to his medication regimen at this time? A. Carvedilol B. Digoxin C. Hydralazine D. Spironolactone

A. Carvedilol Digoxin (B) is medication used in heart failure with reduced ejection fraction and supraventricular dysrhythmias. It's mechanism of action in systolic heart failure is inhibition of the sodium-potassium ATPase pump in myocardial cells, resulting in a transient increase in intracellular sodium, which promotes calcium influx, leading to increased contractility. Digoxin also suppresses AV node conduction. Digoxin is considered a secondary pharmacologic therapy in heart failure with reduced ejection fraction (the combination of diuretics, angiotensin system blockers, and beta-blockers are the initial and primary pharmacologic therapies). Digoxin is added among patients with persistent moderate to severe symptoms for initial therapy plus all other indicated secondary therapies. Digoxin does not decrease mortality but has shown improvement in symptoms, including reduction in hospitalization for heart failure. Hydralazine (C) is another secondary medication used in heart failure with reduced ejection fraction as well as hypertension. It works by vasodilating arterioles and decreasing systemic resistance. Hydralazine in combination with nitrate is an alternative regimen among patients with heart failure with reduced ejection fraction who are intolerant of ACE inhibitors, ARBs, and ARNIs. Moreover, the combination of hydralazine with a nitrate has shown a mortality benefit among Black men as compared to White men, so the combination of these two medications is often added to medication regimens of Black patients with heart failure with reduced ejection fraction. Spironolactone (D) is another secondary agent that may be used in heart failure with reduced ejection fraction after patients have been optimized on a diuretic, ACE inhibitor, and beta-blocker. Spironolactone is a potassium-sparing diuretic and antihypertensive. It increases sodium chloride and water excretion while conserving potassium and hydrogen ions. It is considered in patients with heart failure with reduced ejection fraction and a left ventricular ejection fraction ≤ 35% who have persistent symptoms on initial therapy (generally a diuretic, beta-blocker, and ACE inhibitor, ARB, or ARNI).

A 35-year-old man presents with complaints of swelling and pain over his left knee for the past 2 days. He lays carpet for a living, and he states he has been on his knees more lately. The patient reports he had a steroid injection into his knee due to pain about 3 days ago. The patient has no chronic medical conditions, does not take any medications, and has no known allergies. Vital signs include BP 120/80 mm Hg, pulse 70 beats per minute, respiratory rate 16 breaths per minute, temperature 98.6°F, and O2 saturation of 98%. Physical exam reveals localized warmth, tenderness, fluctuant edema, and erythema localized over the patella. Passive joint range of motion is preserved. Fluid aspiration analysis shows WBC count of 3,500/microL and a Gram stain shows gram-positive cocci. Aerobic and anaerobic cultures are pending. Which of the following is the best therapy for this patient, given the suspected diagnosis? A. Cephalexin B. Nafcillin C. Trimethoprim-sulfamethoxazole D. Vancomycin

A. Cephalexin Septic bursitis is inflammation of a bursa due to infection that most commonly occurs at the olecranon, prepatellar, and infrapatellar bursae. Risk factors include trauma, preceding noninfectious bursa inflammation such as from gout or rheumatoid arthritis, loss of skin integrity from underlying dermatologic conditions, recent joint aspiration or injection, male sex, and immunosuppression. It is most commonly caused by Staphylococcus aureus. Patients typically present with acute onset of tenderness, pain, edema, erythema, and warmth at the site of the bursa. Physical exam findings include fever, warmth of bursa, tenderness to palpation, fluctuant edema, and localized erythema. Passive joint range of motion is preserved when infection is isolated to the bursa. Patients with limitation of joint movement should be evaluated for concomitant septic arthritis. Patients with prepatellar or infrapatellar septic bursitis may also develop an aseptic knee effusion. In this case, fluid aspiration typically reveals a low white blood cell count. Evaluation includes aspiration of bursa fluid and X-ray of the affected joint. X-ray is used to evaluate for local trauma and osteomyelitis. Bursa fluid should be sent for Gram stain, aerobic and anaerobic culture, white blood cell count with differential, and evaluation for crystals. Typical bursa fluid findings include leukocyte count greater than 1,000-3,000 cells/microL. Gram stain can be positive or negative. The gold standard of diagnosis is bursa fluid culture, but culture can be negative due to recent antibiotic use or infection with fastidious organisms. Initial management is with antibiotic therapy and serial evaluation by an orthopedic surgeon. Surgical drainage is indicated for patients with signs of mass effect from a severely swollen bursa (diminished perfusion or compression neuropathy) or persistent symptoms after 36-48 hours of antimicrobial therapy. Patients with systemic signs of toxicity (fever greater than 100.5°F and tachycardia), bursitis overlying prosthetic joint or graft, rapid progression of erythema, progression after 24 hours of oral antibiotics, increased risk of methicillin-resistant Staphylococcus aureus (MRSA), prior history of MRSA, or inability to tolerate oral therapy should be admitted for IV antibiotics and surgical consultation. Risk factors for MRSA include hospitalization in the last 60 days, residence in a long-term care facility, surgery in the last 60 days, hemodialysis, and HIV infection. Patients who are hospitalized and are at risk for MRSA infection should be treated with IV vancomycin and cefazolin. IV cefazolin can be used for suspected methicillin-susceptible Staphylococcus aureus (MSSA) and Streptococcus infections. Patients without systemic signs of toxicity can be treated with oral antibiotic therapy. Trimethoprim-sulfamethoxazole is used for suspected MRSA infections and cephalexin or dicloxacillin is used for MSSA or Streptococcus infections. In general, at least 10 days of antibiotics are warranted. This patient did not present with any risk factors for MRSA and can, therefore, be treated with coverage for MSSA. Nafcillin (B) is recommended for patients with MSSA or Streptococcus infection that meets the above criteria for intravenous antibiotic therapy. This patient does not present with any signs of systemic toxicity and so can be treated with outpatient antibiotics. Trimethoprim-sulfamethoxazole (C) is recommended for patients who do not meet the criteria for IV antibiotic therapy but where there is a concern for MRSA infection. This patient does not present with any risk factors concerning for MRSA infection. Vancomycin (D) is recommended for patients with an increased risk of MRSA infection or suspected MRSA infection who have a fever greater than 100.5°F and tachycardia, bursitis overlying prosthetic joint or graft, rapid progression of erythema, progression after 24 hours of oral antibiotics, or who cannot tolerate oral therapy.

A 28-year-old man presents with fatigue and weight gain. He states he has difficulty concentrating on tasks at work and frequently has feelings of worthlessness. He has difficulty both falling asleep and maintaining sleep. He has lost interest in social interaction and does not enjoy the activities that he once enjoyed. These symptoms have been occurring for the last 4 weeks. Which of the following historical findings would be most consistent with the diagnosis? A. Childhood trauma B. Family history of bipolar disorder C. High birth weight D. Tobacco use

A. Childhood trauma A family history of bipolar disorder (B) increases the risk of developing bipolar depression but not unipolar depression or major depressive disorder. High birth weight (C) is not associated with major depressive disorder, but low birth weight has been correlated with a greater risk of developing depression later in life. Tobacco use (D) does not increase the risk of depression, but substance use disorder is associated with major depressive disorder.

A 42-year-old woman presents to her primary care office, reporting symptoms of fatigue, intermittent palpitations, slight tremors, and heat intolerance over the past 3-4 months. Her weight has dropped 20 pounds since her last office visit 4 months ago. The patient notes she has not changed her eating habits and has not been exercising very much lately. On physical exam, her thyroid is diffusely enlarged and there is bilateral exophthalmos present. A full thyroid panel is ordered on her blood work, and she is noted to have a thyroid disorder. Which of the following lab values would be most consistent with this patient's diagnosis? A. Decreased TSH, elevated T4, elevated T3 B. Decreased TSH, normal T4, elevated T3 C. Elevated TSH, decreased T4, decreased T3 D. Elevated TSH, elevated T4, elevated T3

A. Decreased TSH, elevated T4, elevated T3 Hyperthyroidism Sx: heat intolerance, palpitations, weight loss, tachycardia, and anxiety PE: hyperreflexia, goiter, exophthalmos, pretibial edema Labs: low TSH and high free T4 Most commonly caused by Graves disease (autoimmune against TSH receptor) Tx: methimazole or PTU PTU in the first trimester of pregnancy Decreased TSH, normal T4, elevated T3 (B) is most consistent with the diagnosis of T3 toxicosis. Elevated TSH, decreased T4, decreased T3 (C) is consistent with the diagnosis of primary hypothyroidism. Elevated TSH, elevated T4, elevated T3 (D) is consistent with the diagnosis of a TSH-producing pituitary adenoma.

A 59-year-old man presents to his primary care provider with increasing dyspnea on exertion and fatigue for the past 3 months. The patient states he has had to sleep in his recliner because he has difficulty breathing and occasionally wakes up gasping for air when he lies flat on his bed. Cardiac and pulmonary auscultations are unremarkable. Bilateral 1+ pitting edema is noted to the distal extremities. Which of the following is the next step to confirm the suspected diagnosis? A. Echocardiography B. Electrocardiogram C. Natriuretic peptide level D. Right heart catheterization

A. Echocardiography An electrocardiogram (B) and natriuretic peptide levels (C) are both part of the initial workup for heart failure but are not sufficient as individual tests to confirm the diagnosis of heart failure. A right heart catheterization (D), although considered the gold standard, is rarely used to diagnose heart failure because it is an invasive test. An echocardiogram is noninvasive and in most cases is sufficient to make the diagnosis.

A 60-year-old man with a 35 pack-year history of smoking presents to your office with complaints of shortness of breath, chronic cough, and sputum production, worsening over the past year. Physical exam reveals decreased breath sounds and wheezes on auscultation of the lungs and pulse oximetry of 93%. Which of the following would be seen on spirometry? A. FEV1/FVC ratio decreased, not reversing to normal with bronchodilator B. FEV1/FVC ratio decreased, reversing to normal with bronchodilator C. FEV1/FVC ratio normal with FEV1 > 80 percent predicted D. FEV1/FVC ratio normal with FVC 70-80 percent predicted

A. FEV1/FVC ratio decreased, not reversing to normal with bronchodilator Chronic Obstructive Pulmonary Disease (COPD) Main subtypes: chronic bronchitis, emphysema Hypoxemic patients should be given supplemental oxygen with SaO2 goal of 88-92% Treatment options include bronchodilators, anticholinergics, steroids, supplemental oxygen, noninvasive ventilation, smoking cessation When the FEV1/FVC ratio is decreased, this reveals a limitation of airflow as seen in both COPD and asthma. Asthma is differentiated from COPD by whether there is reversal to normal with bronchodilator (B), as seen in asthma, or no reversal to normal, as seen in COPD. FEV1/FVC ratio normal with FEV1 > 80 percent predicted (C) is a normal spirometry reading seen in patients without lung disease. Individuals undergoing spirometry who have a normal FEV1/FVC ratio with FVC 70-80 percent predicted (D) see this result due to either poor technique or obese body habitus.

A 28-year-old woman presents to the gastroenterology clinic for reevaluation of painless rectal bleeding with bowel movements. She has no personal or family history of colorectal cancer or inflammatory bowel disease. On physical exam, she has visible hemorrhoids originating below the dentate line. Her hemoglobin level is 14.0 mg/dL. She has been treating her condition with increased water intake, increased dietary fiber, sitz baths, and docusate for 12 weeks without improvement. Which of the following is the recommended treatment? A. Hemorrhoidectomy B. Rubber band ligation C. Sclerotherapy D. Topical hydrocortisone

A. Hemorrhoidectomy Hemorrhoids Sx: discomfort and itching in the anal region, if thrombosed may also report pain PE: Internal: proximal to the dentate line External: distal to the dentate line Tx: lifestyle modifications, sitz baths, analgesic creams, rubber band ligation, sclerotherapy, surgical excision Rubber band ligation (B) is a procedure performed in office on patients with grade I-II internal hemorrhoids who fail to improve with 6-8 weeks of conservative treatment and in patients with grade III internal hemorrhoids. The grading of internal hemorrhoids is based on the amount of prolapse. Grade I hemorrhoids do not protrude through the anus, grade II protrude and then reduce spontaneously, and grade III must be manually reduced. Rubber band ligation cannot be performed on external hemorrhoids because they are sensitive to pain. Sclerotherapy (C) involves injecting sclerosant solutions to treat symptomatic internal hemorrhoids. In sclerotherapy, the sclerosant causes an inflammatory reaction, which destroys excess submucosal tissue. Sclerotherapy is performed through an anoscope and does not require anesthesia (local or general). Sclerotherapy can be used to treat grade I-II bleeding internal hemorrhoids. Topical hydrocortisone (D) is one of the topical medications used to treat patients with pain or pruritus related to hemorrhoids. Topical hydrocortisone is part of the conservative treatment, but this patient did not respond to conservative treatment.

A 32-year-old woman develops a right lower extremity deep vein thrombosis and is placed on anticoagulation therapy. She is a smoker and was using an oral estrogen and progestin combination pill for contraception prior to developing the deep vein thrombosis. She is an otherwise healthy individual without any chronic medical issues. Which of the following would be the best method of birth control for this patient in the future? A. Progestin-only oral contraceptive B. Spermicidal vaginal suppositories C. Transdermal contraceptive patch D. Vaginal ring

A. Progestin-only oral contraceptive Spermicidal vaginal suppositories (B) are not as effective at preventing pregnancy and not as convenient as a progestin-only oral contraceptive. The transdermal contraceptive patch (C) and the vaginal ring (D) both include estrogen. Estrogen should be avoided permanently in those with a history of thromboembolic events to reduce the risk of further events occurring. Estrogen and progestin combination contraceptives should be avoided in women who are at an increased risk of thromboembolic events. These risk factors include age above 35 years, smokers, those with recent leg trauma, and those with sedentary lifestyles. A progestin-only contraceptive is a good choice for contraception in patients who cannot tolerate estrogen. These medications consist of norethindrone or levonorgestrel and are effective without suppressing ovulation. These contraceptives work by making the cervical mucus less permeable and the endometrial lining less hospitable for implantation. The progestin-only pill is taken every day without any placebo, which makes it less likely to be missed and increases medication compliance. Side effects include irregular vaginal bleeding and weight gain.

62-year-old man presents to the office with a 2-month history of fatigue, shortness of breath, and intermittent headaches. His physical examination is pertinent for conjunctival pallor, dried blood in the nares, and ecchymosis on the shins and arms. No lymphadenopathy or organomegaly is appreciated. A complete blood count with differential is obtained and reveals pancytopenia. Peripheral smear shows the above cell. What is the most likely diagnosis? A. Acute lymphoid leukemia B. Acute myeloid leukemia C. Chronic lymphoid leukemia Question: What are the metabolic abnormalities associated with tumor lysis syndrome? D. Chronic myeloid leukemia

B. Acute myeloid leukemia Acute Myeloid Leukemia (AML) Patient will be an adult Fever, fatigue, anemia, easy bruising or bleeding, petechiae, bone and joint pain, and persistent or frequent infections PE will show hepatosplenomegaly Labs will show Auer rods, normocytic, normochromic anemia, blast cells Acute lymphoid leukemia (A) is a common malignancy in children. Fever, lymphadenopathy, and organomegaly are common. Lymphoblasts, rather than myeloblasts, are observed. Patients with chronic lymphoid leukemia (C) have lymphocytosis, fever, weight loss, lymphadenopathy, and organomegaly. Chronic myeloid leukemia (D) has a triphasic course associated with chronic, accelerated, and blast phases. Many patients are initially asymptomatic before developing symptoms similar to that of acute myeloid leukemia. Chronic myeloid leukemia is associated with the BCR-ABL1 fusion gene that results in the Philadelphia chromosome.

A 67-year-old man with a history of heart failure, hypertension, and diabetes mellitus presents to your office with complaints of heart palpitations, dizziness, and fatigue. Physical exam findings include a body mass index of 35.2 kg/m2 and an irregularly irregular pulse with a rate of 120 bpm. Electrocardiogram reading shows absent P waves, R-R intervals without a repetitive pattern, and irregular QRS complexes. Which of the following is the most appropriate therapy? A. Amiodarone B. Apixaban C. Furosemide D. Lisinopril

B. Apixaban Cardiac dysrhythmias can present as either asymptomatic or symptomatic. Of the different dysrhythmias, atrial fibrillation is the most common chronic type. There are four types of atrial fibrillation: paroxysmal, persistent, long-standing persistent, and permanent. The condition typically starts as paroxysmal and then progresses to the other types as it becomes an established rhythm. A number of triggers can lead to atrial fibrillation, including thyroid disorders, pericarditis, trauma to the chest, obstructive sleep apnea, cardiac surgery, pulmonary disorders, and certain medications. Excess alcohol or withdrawal from alcohol can also trigger atrial fibrillation, a condition often referred to as holiday heart. Patients with atrial fibrillation may be asymptomatic or present with a range of symptoms, including heart palpitations, dizziness, lightheadedness, fatigue, weakness, dyspnea, or syncope. In more serious cases, individuals may present with thromboembolism or onset of heart failure, as evidenced by pulmonary and peripheral edema, ascites, and weight gain. Workup for atrial fibrillation includes a careful history to determine any reversible causes of the condition, physical examination, laboratory testing, and cardiac testing with electrocardiogram. Cardiac exam may reveal contributing conditions, such as the murmur of mitral stenosis or evidence of heart failure. Laboratory testing is used to rule out thyroid disorders and diabetes mellitus. Prevention of the most serious consequence of atrial fibrillation is critical. Thrombus formation related to stasis in the atria and then embolization to the cerebral circulation resulting in cerebrovascular accident is a consequence that must be addressed in all patients found to have atrial fibrillation. A system referred to as the CHA2DS2-VASc score provides guidance about which patients should receive oral anticoagulation to prevent stroke. Points are given for the presence of heart failure, hypertension, age 65-74 years or > 75 years, diabetes mellitus, stroke, transient ischemic attack or thromboembolism, vascular disease, and female sex. Anticoagulation should be considered when the CHA2DS2-VASc score is 1 and is recommended when the score is ≥ 2. This patient has a CHA2DS2-VASc score of 4 due to his age and the presence of three comorbidities, so he should be started on an anticoagulant. Anticoagulation may be with warfarin, which requires frequent monitoring, or one of four direct oral anticoagulants: dabigatran, rivaroxaban, apixaban, or edoxaban. Patients with atrial fibrillation also typically require slowing of the ventricular rate to improve symptoms. Agents used for rate control include beta-blockers, calcium-channel blockers, and digoxin. Amiodarone (A) is a class III antidysrhythmic agent used in patients with structural heart disease to regulate heart rate. It is sometimes used in patients as part of the management of atrial fibrillation. Amiodarone interacts with both warfarin and digoxin and thus requires careful monitoring of medication levels to ensure patient safety. Furosemide (C) is a loop diuretic that is used in the treatment of congestive heart failure. Patients with heart failure and atrial fibrillation may be on this medication, but it is not used specifically to treat the dysrhythmia symptoms of atrial fibrillation. Lisinopril (D) is an angiotensin-converting enzyme inhibitor used in the treatment of hypertension.

A 6-year-old boy presents to the clinic with right eye redness, yellow-green discharge, and right eye being stuck shut in the mornings. He reports no changes in his vision or eye trauma. On exam, he has injection of the right conjunctiva and yellow discharge in the medial canthus. There is no palpable lymphadenopathy. The left eye is unremarkable. Which of the following is the most likely diagnosis? A. Allergic conjunctivitis B. Bacterial conjunctivitis C. Infectious keratitis D. Viral conjunctivitis

B. Bacterial conjunctivitis The treatment of bacterial conjunctivitis is with antibiotic eye drops or ointments. Examples of ophthalmic eye drops used in the treatment of bacterial conjunctivitis include trimethoprim-polymyxin B and azithromycin. Examples of ophthalmic ointments include erythromycin and sulfacetamide. Fluoroquinolones, such as moxifloxacin drops, are preferred in patients who wear contact lenses. There are no antiviral ophthalmic medications for viral conjunctivitis. If viral conjunctivitis is suspected, then patients should be offered topical ophthalmic antihistamines or decongestants, such as ketotifen. The treatment for allergic conjunctivitis also consists of topical antihistamines or decongestants. Allergic conjunctivitis (A) is due to allergens contacting the eye and causing the release of histamine and other chemical mediators by mast cells. Patients typically have bilateral eye redness, watery discharge, and eye itching. Itchiness is the predominant symptom of allergic conjunctivitis. The patient in this vignette had unilateral involvement and yellow drainage, which are suggestive of an infectious etiology rather than allergic conjunctivitis. Infectious keratitis (C) is an infection of the cornea that is commonly seen in contact lens wearers. The classic symptoms include eye pain, photophobia, and foreign body sensation. Visual acuity may be decreased. The patient in this vignette had symptoms and signs more consistent with conjunctivitis. Viral conjunctivitis (D) classically presents with eye injection, burning and gritting sensation, and watery or mucoserous drainage. Viral conjunctivitis most often begins unilaterally and then affects the contralateral eye within 24-48 hours. Viral conjunctivitis may cause an enlarged or tender preauricular lymph node. Bacterial conjunctivitis can be distinguished from viral conjunctivitis by its unilateral symptoms, purulent discharge, and absence of preauricular lymphadenopathy.

A 28-year-old man presents to the office with concerns of progressively worsening scrotal pain over the past 2 days. He reports associated dysuria beginning today. He reports having vaginal intercourse with women. He reports no anal intercourse. On physical examination, the superior-posterior aspect of the right scrotum is tender to palpation and appears swollen. A positive Prehn sign is noted. What is the most likely etiologic agent for the patient's suspected diagnosis? A. Actinomyces israelii B. Chlamydia trachomatis C. Escherichia coli D. Mycoplasma pneumoniae

B. Chlamydia trachomatis Epididymitis Patient presents with gradual-onset unilateral scrotal pain PE will show increased color flow on Doppler, relief with testicular elevation (Prehn sign) Most commonly caused by < 35 years old: C. trachomatis, N. gonorrhoeae > 35 years old: E. coli, Pseudomonas Treatment < 35 years old: ceftriaxone-doxycycline > 35 years old: fluoroquinolones Actinomyces israelii (A) is commonly associated with pelvic infections in women who have intrauterine devices. Escherichia coli (C) would be the correct answer choice in a patient who is 35 years or older or who is at risk of enteric pathogens due to insertive anal intercourse. Mycoplasma pneumoniae (D) is not commonly implicated in epididymitis, although Mycoplasma genitalium can be. In young boys (under the age of 14), epididymitis can occur as a part of a postinfectious syndrome secondary to respiratory infections secondary to Mycoplasma pneumoniae. This would be unlikely in the patient in the vignette, given his age and risk factors for Chlamydia trachomatis and Neisseria gonorrhoeae species.

A 26-year-old woman presents to her primary care provider with complaints of amenorrhea for 8 weeks. She reports no previous history of amenorrhea, metrorrhagia, nausea, vomiting, or abdominal pain. Physical examination reveals obesity, normal heart and lung sounds, positive bowel sounds in all quadrants, and a soft, nontender, nondistended abdomen without rebound or guarding. Her urine pregnancy test is positive, and her ultrasound confirms an intrauterine pregnancy. An incidental finding on ultrasound is shown above. Which of the following is the most likely diagnosis? A. Choledocholithiasis B. Cholelithiasis C. Chronic cholecystitis D. Porcelain gallbladder

B. Cholelithiasis Choledocholithiasis (A) may be asymptomatic or present with biliary pain, jaundice, and fever. The patient's ultrasound shows gallstones within the gallbladder, but there is no evidence of dilated ducts, which would be seen with choledocholithiasis. Chronic cholecystitis (C) presents with nausea and constant epigastric or right upper quadrant pain. Porcelain gallbladder (D) is a potential complication of chronic cholecystitis and a risk factor for gallbladder cancer. A partially or fully calcified gallbladder would be seen on ultrasound.

A 42-year-old man presents to his primary care provider with complaints of hearing loss and painless ear drainage in his right ear for 1 month. He states that, 2 months ago, he spent a day practicing at the shooting range without any ear protection. Weber test lateralizes to the right ear. Rinne test on the right ear reveals bone conduction greater than air conduction. Otoscopic examination reveals the presence of a pearly white mass in the middle ear. Which of the following is the most likely diagnosis? A. Cerumen impaction B. Cholesteatoma C. Malignant external otitis D. Otitis media with effusion

B. Cholesteatoma Cerumen impaction (A) results in conductive hearing loss and may be present with cholesteatoma. The patient's recent history of trauma and painless otorrhea is more consistent with cholesteatoma. Furthermore, cerumen impaction occludes the ear canal and obscures the tympanic membrane. Malignant external otitis (C) is a serious infection that starts in the outer ear and spreads to the middle ear. It is seen most commonly in older patients with diabetes. The most common causative agent of malignant external otitis is Pseudomonas. Untreated cases can result in the spread of infection and damage to the skull, cranial nerves, and brain. Otitis media with effusion (D) is a noninfectious collection of fluid in the middle ear space behind the tympanic membrane. It may result from an upper respiratory infection or pharyngitis.

A 55-year-old man with a past medical history of hypertension and cigarette smoking presents to the clinic reporting bilateral lower limb pain for the past several months. He states the pain used to only occur when going on long walks, but for the past few weeks, he has had the pain when he walks to the mailbox. Physical exam reveals diffuse hair loss of the bilateral lower extremities and diminished posterior tibial and dorsalis pedis pulses bilaterally. Ankle-brachial index testing is performed in the clinic and reveals an ankle-brachial index of 0.8. The patient is counseled on smoking cessation and lifestyle changes, including exercise therapy. He is seen at a 3-month follow-up visit with minimal improvement in symptoms and no improvement in ankle-brachial index. Which of the following is the most appropriate next step in management? A. Apixaban B. Cilostazol C. Coumadin D. Heparin

B. Cilostazol Peripheral Artery Disease Sx: pain in the affected extremity related to activity (intermittent claudication) PE: cool extremity with absent or diminished pulses Dx: ankle-brachial index (ABI) ≤ 0.9 If limb is threatened: contrast arteriography (gold standard) Caused by atherosclerotic disease Risk factors: smoking, DM, HTN Management: risk factor modification, supervised exercise therapy Apixaban (A) and coumadin (C) are both anticoagulant medications more appropriately used as maintenance to avoid clotting in chronic conditions such as atrial fibrillation. Heparin (D) is indicated in acute conditions requiring resolution of a blockage or clot, such as acute arterial embolism.

A 55-year-old woman presents to the emergency department with sudden-onset shortness of breath and pleuritic chest pain. Her past medical history is significant for a right ankle fracture 2 weeks ago. She is on hormone replacement therapy for menopausal symptoms. Vital signs are BMI 39.2 kg/m2, T 98.6°F, BP 148/88 mm Hg, HR 110 bpm, RR 23 breaths per minute, and oxygen saturation 88% on room air. Her chest X-ray is normal. Forced dorsiflexion of the right foot elicits pain in the right calf. Which of the following diagnostic studies is the best next step for the most likely diagnosis? A. Catheter pulmonary angiography B. Computed tomography pulmonary angiography C. Impedance plethysmography D. Ventilation-perfusion scan

B. Computed tomography pulmonary angiography Catheter pulmonary angiography (A), historically, is the gold standard for diagnosis of pulmonary embolism. It is an invasive test and now is typically only used when the patient will need an endovascular intervention. Impedance plethysmography (C) may be used in serial examinations of patients with negative leg studies though it is slightly less accurate than venous ultrasonography. Impedance plethysmography compares electrical impedance between patent and obstructed veins to identify the presence of a thrombus. A ventilation-perfusion scan (D) that detects a perfusion defect in the presence of normal ventilation is indicative of pulmonary embolism. It is useful in patients who are allergic to the contrast used for computed tomography pulmonary angiography or who have kidney disease.

A 19-year-old G2P1 woman at 12 weeks gestation presents to her primary care provider with complaints of worsening pustules on her face. Physical examination reveals the presence of cystic, inflammatory lesions on the face, neck, upper chest, and back. Which of the following treatments, along with a benzoyl peroxide wash, is most appropriate at this time? A. Doxycycline B. Erythromycin C. Isotretinoin D. Tretinoin

B. Erythromycin Acne Vulgaris More common in adolescents, may occur at other ages PE: open comedones (blackheads), closed comedones (whiteheads), papules, and pustules found on the face, chest, upper back Etiology: sebum production, follicular keratinization, Cutibacterium acnes (formerly Propionibacterium acnes) Treatment may include: benzoyl peroxide +/- topical retinoids or topical antibiotics, oral antibiotics (doxycyline/minocycline) + benzoyl peroxide or retinoid, OCPs, spironolactone Comments: oral isotretinoin is teratogenic (pregnancy class X, must have two forms of birth control) Doxycycline (A), isotretinoin (C), and tretinoin (D) are all contraindicated in pregnancy. Doxycycline is a tetracycline antibiotic. Tetracyclines are contraindicated in pregnancy, as they are associated with tooth discoloration and decrease bone growth. Retinoids such as isotretinoin and tretinoin are associated with an increased risk of birth defects, miscarrage, and preterm delivery. Patients taking isotretinoin must enroll in the iPledge monitoring program, receive regular pregnancy tests, and use two forms of contraception.

A 7-year-old boy presents to the pediatric clinic with a focal red and painful bump on his right upper eyelid. Neither the patient nor his mother report any trauma to his eye, and he first noticed it when he woke up this morning. Which of the following describes the most likely condition? A. Diffuse inflammation of the eyelid B. Focal purulent inflammatory process of the eyelid C. Increased pressure within the eye D. Inflammation of the conjunctiva of the eye

B. Focal purulent inflammatory process of the eyelid Diffuse inflammation of the eyelid (A) is called blepharitis, which is often associated with rosacea or seborrheic dermatitis. Blepharitis presents with bilateral eyelid erythema, edema, and pain. Increased pressure within the eye (C) is called glaucoma. There are acute angle-closure glaucomas and open-angle glaucomas. Angle-closure glaucoma is marked by a sudden and rapid rise in intraocular pressure. Inflammation of the conjunctiva of the eye (D) is called conjunctivitis and may be caused by bacterial, viral, or allergic causes.

A 61-year-old woman with a history of alcohol use disorder and Crohn disease presents to the office for her well-woman visit with concerns of fatigue and feeling mentally foggy. On physical examination, she appears mildly pale, and loss of papillae is noted on the tongue. Baseline blood work is obtained. Her hemoglobin and hematocrit are mildly decreased at 11 g/dL and 32%, respectively. Her mean corpuscular volume is 117 fL. Vitamin B12 and folate levels are borderline low, but methylmalonic acid and homocysteine are elevated. What physical exam finding would be expected on examination given the patient's history and laboratory results? A. Chlorosis B. Gait ataxia C. Koilonychia D. Oral ulcers

B. Gait ataxia Vitamin B12 (Cobalamin) Deficiency Risk Factors: vegan diet, metformin use Sx: fatigue, weakness, and peripheral neuropathy PE: pallor and glossitis Labs: MCV > 100 fL, hypersegmented neutrophils, elevated homocysteine, elevated methylmalonic acid Tx: parenteral vitamin B12, oral considered if no neurologic sx or malabsorption Neuropathy is more common with vitamin B12 deficiency (as opposed to folate deficiency) Pernicious anemia: autoimmune destruction of cells that produce intrinsic factor (IF), resulting in vitamin B12 deficiency Chlorosis (A) refers to a pale green discoloration of the skin observed in severe iron deficiency anemia. This is a rare finding that was more commonly observed in the 1800s. Koilonychia (C) refers to the physical exam finding of spoon nails more commonly observed in iron deficiency anemia. Oral ulcers (D) may be observed in folate deficiency anemia but are not commonly associated with vitamin B12 deficiency anemia. Patients with folate deficiency do not have elevations in their methylmalonic acid. B12 deficiency is more likely in this patient due to her history of Crohn disease, given the disease's predilection for the terminal ileum where vitamin B12 is absorbed.

A 22-year-old woman presents to the office for evaluation. She was in the office 3 days ago for her first urinary tract infection and was prescribed an antibiotic that she had never received before. She has been taking the antibiotic as prescribed but is now experiencing fatigue and yellowing of the skin. Her urine has also become dark in appearance. She questions whether the antibiotic is working or the infection is worsening. On physical examination, she has left upper quadrant abdominal tenderness. Sclera are mildly icteric. Peripheral smear is shown above. What is the most likely diagnosis? A. Allergic interstitial nephritis B. Glucose-6-phosphate dehydrogenase deficiency C. Paroxysmal nocturnal hemoglobinuria D. Pyelonephritis

B. Glucose-6-phosphate dehydrogenase deficiency Allergic interstitial nephritis (A) is an inflammatory reaction with the kidney that occurs secondary to drug therapy, usually antibiotics. The common triad is rash, fever, and eosinophilia. Paroxysmal nocturnal hemoglobinuria (C) is a rare disorder associated with a PIGA gene mutation on the X chromosome that results in episodic hemolytic anemia manifested as hemoglobinuria, particularly at nighttime and in the early morning hours. Patients commonly report fatigue, but jaundice can also occur. Heinz bodies would not be observed. Patients are at increased risk of thrombosis. Patients with pyelonephritis (D) experience fever, flank pain, costovertebral angle tenderness, and nausea or vomiting.

A 45-year-old man with a history of diabetes and newly diagnosed active pulmonary tuberculosis presents to the office for evaluation of numbness and tingling in his feet. He describes the sensation as "pins and needles." Which medication used to treat active tuberculosis is the most likely culprit of this patient's symptoms? A. Ethambutol B. Isoniazid C. Pyrazinamide D. Rifampin

B. Isoniazid Ethambutol (A) is often used in the treatment of active tuberculosis. It inhibits arabinosyl transferase, resulting in impaired mycobacterial cell wall synthesis. Ocular side effects are common, including decreased visual acuity, color blindness (especially decreased red-green color discrimination), and optic neuritis. Pyrazinamide (C) is often used in the treatment of active tuberculosis along with rifampin, isoniazid, and ethambutol. It is converted to pyrazinoic acid in susceptible strains of Mycobacterium, which lowers the pH of the environment. Common side effects include hepatotoxicity, gout flares, and arthralgias. Neuropathy is generally not a side effect. Rifampin (D) is used in the treatment of latent and active tuberculosis. It inhibits bacterial RNA synthesis and blocks RNA transcription. Side effects include hepatotoxicity, flu-like syndromes, and orange discoloration of urine, tears, and sweat. It is also a potent P450 inducer, which means it has many drug interactions. Neuropathy is not a side effect.

A 60-year-old man with a history of hypertension and hyperlipidemia presents to the office for evaluation of new-onset intermittent chest pain. You suspect a diagnosis of coronary artery disease. Which of the following features in his history would support this diagnosis? A. Acute sharp chest pain described as ripping in quality B. Left-sided exertional chest pain that radiates down the left arm C. Pleuritic chest pain accompanied by dyspnea and cough D. Sharp pleuritic chest pain that improves with sitting up and leaning forward

B. Left-sided exertional chest pain that radiates down the left arm Acute sharp chest pain described as ripping in quality (A) is concerning for an aortic dissection. Pain may or may not be associated with syncope, symptoms or signs of stroke, myocardial infarction, heart failure, or other clinical signs of end-organ ischemia. Aortic dissection is a surgical emergency. Pleuritic chest pain accompanied by dyspnea and cough (C) is concerning for a pulmonary embolism. Patients may also present with signs and symptoms of deep venous thrombosis. In cases of a massive pulmonary embolism, patients may present in shock. Sharp pleuritic chest pain that improves with sitting up and leaning forward (D) is typical of pericarditis. These patients typically also have a pericardial friction rub and widespread ST elevations or PR depressions on electrocardiogram.

A 22-year-old woman who is otherwise healthy presents to the office due to red, raised, and painful bumps to the inside of the thighs, starting about 2 days ago. She works as a physical trainer and typically wears tight-fitting workout leggings and has been sweating in the affected areas due to warmer weather. A physical exam shows multiple erythematous pustules to the inner aspect of the thighs with hair emanating from the center of each pustule. Which of the following is the most likely diagnosis? A. Eosinophilic folliculitis B. Nonbacterial folliculitis C. Pseudofolliculitis D. Pseudomonas folliculitis

B. Nonbacterial folliculitis Folliculitis Inflammation of hair follicle Most commonly caused by S. aureus Pruritus, pain Start with topical antibiotic therapy before oral antibiotics Eosinophilic folliculitis (A) is a type of folliculitis common in patients with AIDS, but this patient is otherwise healthy. Pseudofolliculitis (C) occurs when there is pustular formation caused by ingrown hairs secondary to shaving. The lesions are located to the side of the hair follicles and not in them. It is seen most often in the beard region in men but can also be seen in women in commonly shaved areas. Pseudomonas folliculitis (D) commonly occurs on the trunk in healthy patients and is associated with hot tub use with contaminated water. Patients present with follicular papules and pustules that occur 8 to 48 hours after exposure to contaminated water.

A 31-year-old man presents to his primary care provider with complaints of back pain for the past 2 weeks. Which of the following is considered an alarming symptom associated with malignancy? A. Decreased pain with hip flexion B. Pain at rest or night C. Pain for more than 3 weeks D. Weight gain

B. Pain at rest or night Decreased pain with hip flexion (A) is a characteristic feature of spinal stenosis and is not an alarming symptom for malignancy. Pain for more than 3 weeks (C) is not considered an alarming symptom, as 90% of low back pain resolves within 6 weeks of treatment. Unexpected weight loss, not weight gain (D) is an alarming symptom for malignancy.

A 70-year-old woman with a history of hypertension and coronary artery disease presents to the office for evaluation of intermittent claudication. Physical examination reveals diminished peripheral pulses. What other physical examination finding would you expect to find? A. Brown pigmentation around the ankles B. Pallor with lower extremity elevation C. Ulceration at the medial malleolus D. Warm lower extremities

B. Pallor with lower extremity elevation Peripheral Artery Disease Sx: pain in the affected extremity related to activity (intermittent claudication) PE: cool extremity with absent or diminished pulses Dx: ankle-brachial index (ABI) ≤ 0.9 If limb is threatened: contrast arteriography (gold standard) Caused by atherosclerotic disease Risk factors: smoking, DM, HTN Management: risk factor modification, supervised exercise therapy

A 75-year-old woman develops acute pain and erythema in the left preauricular area while she is hospitalized following surgery. Vital signs are T of 101.1°F, HR of 108 bpm, BP of 150/92 mm Hg, RR of 20/min, and oxygen saturation of 95% on room air. Physical examination reveals focal tenderness extending from the left preauricular area to the angle of the mandible, and there is purulent drainage from the Stensen duct. Laboratory findings reveal a white blood cell count of 14,000/microL. Which of the following pathogens is the most likely cause? A. Mumps virus B. Staphylococcus aureus C. Streptococcus pneumoniae D. Streptococcus pyogenes

B. Staphylococcus aureus Mumps virus (A) causes a unilateral or bilateral (more common) nonpurulent parotitis. Mumps also has a more gradual onset and viral prodrome, beginning with fever, headache, myalgia, and anorexia. Mumps may be complicated by orchitis or oophoritis and neurologic complications, including meningitis, encephalitis, and deafness. Unvaccinated individuals are at the highest risk for mumps. Streptococcus pneumoniae (C) and Streptococcus pyogenes (D) are each rare causes of suppurative parotitis. Streptococcus pneumoniae frequently causes community-acquired pneumonia and otitis media. Streptococcus pyogenes causes streptococcal pharyngitis. Parotitis Patient will be older History of dehydration or intubation Pain and tenderness with trismus and dysphagia PE will show a firm, erythematous swelling of the pre- and postauricular areas that extends to the angle of the mandible Most commonly caused by Staphylococcus aureus Treatment is intravenous antibiotics

A 38-year-old man with a prosthetic heart valve presents with fever, cough, and rash. His symptoms started approximately 3 months after his prosthetic heart valve placement surgery. On the physical exam, he is noted to have a holosystolic murmur (heard best at the left lower sternal border) and splinter hemorrhages of the fingernail beds. He also has painful, erythematous papular lesions on his palms and soles. A blood culture is obtained. Which of the following organisms is most likely to be isolated? A. Staphylococcus aureus B. Staphylococcus epidermidis C. Streptococcus bovis D. Streptococcus viridans

B. Staphylococcus epidermidis Staphylococcus aureus (A) can cause early-onset prosthetic valve endocarditis, but it is not the most common cause. However, it is the most common cause of endocarditis in intravenous drug users, particularly affecting the tricuspid valve. Streptococcus bovis (C) and Streptococcus viridans (D) are the most frequent causative organisms in cases of late-onset prosthetic valve endocarditis, with symptoms typically appearing more than 12 months after surgery. Streptococcus bovis is highly associated with an increased risk of active colonic malignancy, whereas Streptococcus viridans is also the most common cause of native valve endocarditis and subacute endocarditis. Bacterial Endocarditis Risk factors: injection drug use, valvular heart disease Sx: fever, rash, cough, and myalgias PE: fever, Roth spots, Osler nodes, murmur, Janeway lesions, anemia, nailbed hemorrhages, emboli (FROM JANE) Diagnosis is made by echocardiography and Duke criteria Most commonly caused by: IVDA: Staphylococcus aureus, tricuspid Native valve: Staphylococcus aureus, viridans streptococci (most common in previously diseased), mitral Tx: antibiotics GI malignancy: Streptococcus bovis Dental prophylaxis in some cases

A 13-year-old boy presents to his pediatrician for his first sports physical. Physical examination is significant for expiratory wheezes. Pulmonary function and bronchodilator response tests are ordered. The patient's baseline forced expiratory volume in 1 second is 75% of the predicted value, and his forced vital capacity is 3.5 L. Which of the following bronchodilator responses is most consistent with asthma? A. A decrease in both forced expiratory volume and forced vital capacity B. A decrease in forced expiratory volume and an increase in forced vital capacity C. An increase in both forced expiratory volume and forced vital capacity D. No change in forced expiratory volume or forced vital capacity

C. An increase in both forced expiratory volume and forced vital capacity A decrease in both forced expiratory volume and forced vital capacity (A) may be seen with poor effort but is not expected with bronchodilator testing in any other condition. A decrease in forced expiratory volume and an increase in forced vital capacity (B) is not consistent with any respiratory condition. No change in forced expiratory volume or forced vital capacity (D) is more consistent with COPD, which is an irreversible airway disease.

A 58-year-old woman who is postmenopausal presents to the clinic for an annual exam. She has no complaints, and physical exam is normal. Family history is positive for diabetes mellitus and hypertension but negative for cancer. Which of the following represents appropriate health maintenance for this patient, according to the U.S. Preventive Services Task Force? A. Bilateral breast ultrasound every 2 years B. Bilateral screening mammography and breast ultrasound every 3 years C. Bilateral screening mammography every 2 years D. Bilateral screening mammography every year

C. Bilateral screening mammography every 2 years Bilateral breast ultrasound every 2 years (A) is not recommended as a screening tool for breast cancer. Patients who are younger than 30 years old and present with a breast mass should undergo directed breast ultrasound for evaluation of the mass. Patients with abnormalities on mammography should also undergo directed breast ultrasonography of the area in question. Bilateral screening mammography and breast ultrasound every 3 years (B) is not recommended by any organizations currently. If annual or biennial screening mammography reveals no suspicious lesions, no ultrasound is recommended. Bilateral screening mammography every year (D) is no longer the standard of care according to most organizations, although the American College of Obstetricians and Gynecologists, the American College of Radiography, and the National Comprehensive Cancer Network all recommend annual bilateral screening mammography. The American Cancer Society recommends annual bilateral screening mammography until age 55, at which point the interval of screening may increase to every 2 years.

A 29-year-old woman presents to the clinic complaining of a fainting spell yesterday while at home. She states she got up from the table after studying for some time and, upon rising, felt hot and nauseous and woke up later on the ground. She reports no loss of bowel or bladder control or any subsequent fatigue or confusion. She has no history of head trauma, is taking no medications, and her pregnancy test was negative. This is her first syncopal episode. Vital signs are within normal limits. Which of the following diagnostic studies is essential in the workup of this patient's syncope? A. Basic metabolic panel B. Computed tomography of the head C. Electrocardiogram D. Electroencephalogram

C. Electrocardiogram Syncope is defined as a transient, self-limited loss of consciousness due to inadequate cerebral blood flow. The four main causes of syncope are reflex syncope, orthostatic syncope, cardiac dysrhythmias, and structural cardiopulmonary disease. Symptoms of syncope differ slightly depending on the etiology. Reflex syncope refers to syncope as the result of neurally mediated decreases in blood pressure. Vasovagal, situational, carotid sinus, and micturition syncope are all forms of reflex syncope. Symptoms of reflex syncope include lightheadedness, feeling warm or cold, sweating, palpitations, nausea, decreased vision, and decreased hearing or abnormal auditory sounds (whooshing). Onlookers may report pallor prior to the syncopal episode. Orthostatic syncope often looks like reflex syncope, only in the setting of change of position from lying to sitting or sitting to standing. Causes of orthostatic syncope include certain medications, low baseline blood pressure, and dehydration. Cardiac dysrhythmias and structural cardiopulmonary disease often cause a syncopal syndrome that occurs without warning and in the midst of activity. There is sometimes a family history of sudden death or heart disease. Evaluation of the patient who presents after a syncopal episode should always include a detailed history, a physical exam with attention to the neurologic and cardiopulmonary systems, and an electrocardiogram to rule out dysrhythmias. Patients with suspected orthostatic syncope should have their blood pressure checked while lying down, sitting up, and standing up. Decreases of more than 20 mm Hg systolic or 10 mm Hg diastolic upon rising indicate orthostasis. Patients with suspected cardiopulmonary structural syncope should have an echocardiogram to evaluate any potential valvular or structural abnormalities. Treatment of syncope is dependent on the subtype. Reflex syncope has no definitive treatment and tends to recur. Patients should be warned to avoid triggers, recognize prodromal symptoms, and lie down or pull off the road during prodrome. Certain maneuvers, such as leg crossing, hand clenching, and arm flexing, can sometimes prevent a full syncopal episode. Patients with orthostatic syncope should be advised to increase fluid intake and maintain good hydration, rise slowly, and avoid medications that increase orthostasis. Patients with cardiac dysrhythmias or cardiopulmonary structural abnormalities warrant referral to a specialist.

A 45-year-old woman presents with unilateral facial drooping on her right side. She says it has been getting worse over the past 2 days and is not painful. Physical exam findings include right eyebrow drooping, inability to close the right eye, and drooping of the right corner of the mouth. Which of the following additional physical exam findings would be consistent with the suspected diagnosis? A. Forehead sparing B. Hearing loss C. Loss of taste D. Right arm weakness

C. Loss of taste Bell Palsy History of viral prodrome Waking up with unilateral facial nerve paralysis, hyperacusis, and taste disturbance PE will show CN VII palsy that does NOT spare the forehead Most commonly caused by HSV Treatment is prednisone, artificial tears, tape eyelid shut, antivirals (for severe cases) Bilateral: Lyme disease, infectious mononucleosis Forehead sparing (A) should prompt an evaluation for a central lesion as the cause of the patient's symptoms due to bilateral innervation to the area. Bell palsy affects the forehead resulting in loss of forehead wrinkling and drooping of the eyebrow. Hearing loss (B) with the patient's associated symptoms should prompt evaluation for Ramsay Hunt syndrome. Ramsay Hunt syndrome classically presents with a triad of ipsilateral facial paralysis, ear pain, and vesicles in the ear or on the face or ear. Bell palsy is characterized by hyperacusis, not hearing loss. Right arm weakness (C) should prompt evaluation for other etiologies, such as stroke or brain tumor. Bell palsy is characterized by isolated facial weakness. Bell palsy is the most common form of facial paralysis, and it affects the seventh cranial nerve. It is believed to be caused by a herpes simplex virus activation that leads to nerve inflammation, demyelination, and palsy. Patients present with sudden onset of unilateral facial paralysis. Maximum weakness is typically present by 3 weeks and can be preceded by pain behind the ear for 1 to 2 days. Common physical exam findings include drooping at the affected corner of the mouth, drawing of the mouth to the unaffected side, inability to close the eye, eyebrow sagging, hyperacusis, decreased tearing, loss of taste on anterior two-thirds of the tongue, and disappearance of nasolabial fold. Patients should have a full neurological exam and should be evaluated for rashes or lesions on the face or external ear, which would indicate varicella zoster and a possible alternative diagnosis of Ramsay Hunt syndrome. Forehead sparing should prompt evaluation for a possible central lesion such as a stroke. Diagnosis is typically made on clinical characteristics. Further imaging may be warranted if physical exam findings are atypical, if symptoms progress beyond 3 weeks, and if there is no improvement at 4 months. Testing for Lyme disease is recommended when there is a possibility of exposure. Treatment should be initiated with oral prednisone 60-80 mg/day for 1 week. Ideally, treatment should be initiated within the first 3 days of symptom onset. No proven benefit has been found with the addition of the antiviral agents valacyclovir and acyclovir, however, these agents have been prescribed in addition to prednisone in severe cases. Management should also include ophthalmologic care to prevent damage to the cornea with taping of the eyelid or covering the eye during sleep and artificial tears. Massage of the weakened facial muscles and physical therapy are also beneficial. Prognosis is favorable if some recovery is seen within 21 days, and most patients have some recovery by 4 months. Patients with incomplete paralysis have a better chance of recovering normal function.

A 13-year-old boy presents to his primary care provider complaining of frequent episodes of wheezing and shortness of breath. His mother states over the last few months he has needed his albuterol inhaler daily. His teacher recently notified the patient's mother that he has been sitting out of recess sometimes due to his asthma. He has also been waking up from sleep at least once per week with symptoms. The patient's current asthma treatment is a short-acting beta-agonist, albuterol, as needed and a low-dose inhaled corticosteroid. What is the appropriate therapy for this patient's asthma? A. He is already on appropriate therapy B. High-dose inhaled corticosteroid and a long-acting beta-agonist C. Low-dose inhaled corticosteroid and a long-acting beta-agonist D. Short-acting beta-agonist as needed only

C. Low-dose inhaled corticosteroid and a long-acting beta-agonist He is already on appropriate therapy (A) would be incorrect because the patient is only taking a short-acting beta-agonist as needed and a low-dose inhaled corticosteroid, which is the appropriate therapy for mild persistent asthma. High-dose inhaled corticosteroid and long-acting beta-agonist (B) is the appropriate therapy for severe persistent asthma. Short-acting beta-agonist as needed only (D) is the appropriate therapy for intermittent asthma.

A 48-year-old woman presents to the clinic complaining of copious amounts of diarrhea and intolerable abdominal cramps for the past 2 months. She was started on several medications at her last visit for management of her type 2 diabetes mellitus. Which of the following agents is the most likely cause of her current symptoms? A. Empagliflozin B. Glimepiride C. Metformin D. Pioglitazone

C. Metformin Empagliflozin (A) is a sodium-glucose cotransporter-2 inhibitor used in the treatment of type 2 diabetes mellitus as an adjunct therapy or as monotherapy in patients who are intolerant of metformin. Empagliflozin works by inhibiting reabsorption of filtered glucose by the kidneys' proximal tubules. Consequently, patients taking empagliflozin will have glucosuria. Patients with a glomerular filtration rate under 45 mL/min/1.73 m2 should not take empagliflozin. The most common side effect of sodium-glucose cotransporter-2 inhibitors is urinary tract infection. Other rare but serious side effects include bone fractures, perineal necrotizing fasciitis, and ketoacidosis. Glimepiride (B) is a sulfonylurea drug that stimulates insulin secretion from pancreatic beta cells. Glimepiride is not a first-line choice in the treatment of type 2 diabetes mellitus due to the increased risk for hypoglycemia but is used as an adjunct when therapy with metformin alone is insufficient. Patients taking glimepiride must monitor their blood glucose at home regularly and be advised of the signs and symptoms of hypoglycemia. A potentially serious side effect of glimepiride is an increased risk of cardiovascular mortality. Patients with a sulfa allergy may also be allergic to drugs in the sulfonylurea class. Pioglitazone (D) is in the thiazolidinedione drug class and lowers blood glucose by improving target cell response to insulin. Because receptors in the kidneys are also activated by pioglitazone, a major side effect of the drug is kidney fluid retention and peripheral edema. The thiazolidinediones may also exacerbate heart failure and are contraindicated in patients with class III or IV heart failure.

A 21-year-old man presents with an insidious onset of nonproductive cough for 2 weeks. He reports sore throat, myalgias, and a high fever. Physical exam reveals a normal pulse despite a high fever. Diffuse crackles and rhonchi are noted on auscultation. Laboratory studies show mild leukocytosis but are otherwise unremarkable. His chest radiograph is shown above. Which of the following organisms is the most likely cause of his condition? A. Chlamydophila psittaci B. Legionella C. Mycoplasma pneumoniae D. Streptococcus pneumoniae

C. Mycoplasma pneumoniae Chlamydophila psittaci (A), like M. pneumoniae, can cause atypical pneumonia. However, the majority of C. psittaci cases are associated with bird exposure, and the onset of symptoms is abrupt rather than insidious. Chest radiographic findings may include interstitial or lobar infiltrates. Infection with Legionella spp (B) causes atypical pneumonia either as a sporadic infection or as an outbreak. It is transmitted from person to person by inhalation of aerosol droplets that contain the bacteria. It is more prevalent in smokers and is associated with exposure to aerosol-producing devices (showers, whirlpool spas, grocery shop mist machines), air travel, and proximity to rivers or watersheds. A typical patient would present with a nonproductive cough and predominant gastrointestinal symptoms. Labs often show hyponatremia, and a urinary antigen for Legionella is positive even weeks after treatment has been started. Interstitial infiltrates are seen on chest radiograph. Streptococcus pneumoniae (D) is the most common cause of typical pneumonia. A patient would present with an acute onset of fever, shaking chills, productive cough, pleuritic chest pain, and dyspnea. A physical exam would reveal tachycardia, tachypnea, late inspiratory crackles, increased tactile fremitus, egophony, and percussive dullness. A chest radiograph would demonstrate lobar consolidation, with multilobar consolidation indicating severe disease.

A 32-year-old man presents to the clinic with acute onset of persistent vertigo for the past 2 days. He also complains of nausea, vomiting, hearing loss, and tinnitus. The symptoms are constant. Neurologic examination reveals a horizontal nystagmus that is suppressed with visual fixation, with no focal neurologic deficits. Which of the following is the recommended treatment? A. Alteplase B. Epley maneuver C. Prednisone D. Valacyclovir

C. Prednisone Labyrinthitis Vestibular neuritis + unilateral hearing loss = labyrinthitis History of recent upper respiratory infection Sudden and persistent onset of vertigo and unilateral hearing loss lasting several days to a week Spontaneous horizontal nystagmus Dx is clinical Tx with corticosteroids, symptomatic meds, vestibular rehabilitation Alteplase (A) is also known as tissue plasminogen activator and is a thrombolytic agent used to treat acute ischemic strokes in patients who present within 4.5 hours of the last known asymptomatic time and do not have any contraindications. The patient in this vignette did not have any findings or risk factors suggestive of a central etiology, such as an ischemic stroke. The Epley maneuver (B) is used in the treatment of benign paroxysmal positional vertigo. However, the patient in this vignette had constant symptoms for 2 days, which is not consistent with benign paroxysmal positional vertigo. Benign paroxysmal positional vertigo produces episodic symptoms that are associated with changes in head position. Valacyclovir (D) is an antiviral medication. Although labyrinthitis may be due to viral illnesses, studies have not shown improved outcomes in patients with labyrinthitis treated with antiviral medications.

A 30-year-old woman presents to her primary care physician with concerns about a firm, painless, rubber ball-like mass on the dorsal aspect of her right wrist. She states the mass seems to change in size but does not cause any numbness or tingling. She reports no trauma to her wrist. She reports she has had an increased workload for the past month in her job as a data entry specialist. Physical examination reveals a 1.5 cm well-circumscribed fluid-filled mass over the scapholunate joint that transilluminates with light. The mass is nontender to palpation. Strength and neurovascular testing of the upper extremity are normal. The mass is shown in the image above. Which of the following clinical interventions is most appropriate at this time? A. Closed rupture by force B. Needle aspiration C. Reassurance and observation D. Surgical excision

C. Reassurance and observation Closed rupture by force (A) was previously accomplished by bashing the cyst with a heavy object, such as a Bible, hence the name Bible cyst. However, this method is no longer recommended as a treatment of ganglion cyst. Needle aspiration (B) is the first-line treatment for symptomatic ganglion cysts. The patient in the vignette is not having any symptoms. Needle aspiration is recommended over surgical excision, though recurrence rates are > 50% after aspiration. Surgical excision (D) is reserved for patients who fail conservative treatment.

8-year-old woman presents to the clinic reporting she has been feeling sad, has not wanted to hang out with friends or family, and is having difficulty sleeping at night for the past 5 weeks. The patient reports being raped by a male acquaintance after attending a mutual friend's birthday party just before her symptoms began. Since then, she has had intermittent flashbacks of the rape and has been avoiding any gathering with her friends. Which of the following is the most appropriate medical therapy for this patient in addition to cognitive behavior therapy? A. Amitriptyline B. Duloxetine C. Sertraline D. Venlafaxine

C. Sertraline Amitriptyline (A) is a tricyclic antidepressant, and both duloxetine (B) and venlafaxine (D) are serotonin and norepinephrine reuptake inhibitors (SNRIs). Although these are all considered antidepressants, they are not the first-line treatment for post-traumatic stress disorder. Post-Traumatic Stress Disorder (PTSD) Sx duration > 1 month Persistently reexperiencing of event Persistently ↑ arousal Avoidance of stimuli ↑ risk for suicide, substance use

A previously healthy 22-year-old woman presents to your clinic with a complaint of shortness of breath while exercising. She has a history of participation in cross-country skiing since childhood and only experiences respiratory symptoms with exercise. Which of the following is the most appropriate therapy? A. Inhaled corticosteroids B. Phosphodiesterase inhibitor C. Short-acting beta-agonist use before exercise D. Short-acting beta-agonist use when symptomatic

C. Short-acting beta-agonist use before exercise Inhaled corticosteroids (A) are used as first-line agents to treat persistent asthma. Many patients with exercise-induced asthma are only symptomatic with activity and thus do not require daily inhaler use. The phosphodiesterase inhibitor (B) theophylline is an oral bronchodilator that has been in use since the 1930s. It has a narrow therapeutic window and needs to be monitored closely due to serious dose-related toxicities, such as tachydysrhythmias, seizures, hypokalemia, and hyperglycemia. Because of the risks, it is not first-line treatment for use in persistent asthma and does not play a role in treating exercise-induced asthma. Short-acting beta-agonist use when symptomatic (D) may be needed for patients with exercise-induced asthma but should be done in addition to pre-exercise dosing when symptoms flare. The goal is to prevent symptoms with pre-exercise dosing of short-acting beta-agonist agents and not to wait for symptoms to occur, which can put the individual at risk of status asthmaticus.

A 72-year-old man presents with bloody diarrhea for 3 days. He complains of crampy abdominal pain and eight loose stools tinged with blood per day. The patient reports no dietary changes and states his wife also has similar symptoms. His vital signs are within normal limits, and he takes no daily medications. On exam, the patient has good turgor, diffuse abdominal tenderness to palpation with no rebound or guarding, and hyperactive bowel sounds. Which of the following is the best next step in diagnosing this patient's illness? A. Fecal hemoccult testing B. Stool bacterial culture for Clostridioides difficile C. Stool bacterial culture for Salmonella, Campylobacter, and Shigella D. Stool microscopy for ova and parasites

C. Stool bacterial culture for Salmonella, Campylobacter, and Shigella Fecal hemoccult testing (A) is not necessary for this patient because he reports visible blood in his stools for the past 3 days. In instances where a patient cannot give a good history of the nature and consistency of their stools, fecal hemoccult testing may be beneficial. Stool bacterial culture for Clostridioides difficile (B) is an important part of the evaluation of a patient with diarrhea who is currently in the hospital, has recently been in a hospital setting, or has recently been on a course of antibiotics. Stool microscopy for ova and parasites (D) is important in the evaluation of a patient who has had exposure to contaminated water, who has recently returned from a prolonged stay in a resource-limited country, who has protracted diarrhea and exposure to a daycare setting, or in cases of protracted diarrhea in men who have sex with men.

An 18-year-old man presents with an acute onset of sore throat and fever. He reports no coughing, nasal congestion, and hoarseness. Additionally, he reports no history of oral sex. On exam, tonsillar exudates and anterior cervical lymphadenopathy are noted. A rapid antigen detection assay is positive. Which of the following is the most likely diagnosis? A. Gonococcal pharyngitis B. Infectious mononucleosis C. Streptococcal pharyngitis D. Viral pharyngitis

C. Streptococcal pharyngitis Pharyngitis Most commonly caused by viral > bacteria (GAS, S. pyogenes) Centor criteria: estimates probability pharyngitis is streptococcal based on PE and Sx: cervical lymphadenopathy, tonsillar exudate, fever, absence of cough Treatment Viral: supportive Bacterial: Penicillin (first line) Patients with Penicillin allergy Cephalexin, cefadroxil (avoid in individuals with immediate-type hypersensitivity to penicillin) Clindamycin Azithromycin, clarithromycin (resistance of group A strep to these agents is known well and varies geographically and temporally) Gonococcal pharyngitis (A) occurs in sexually active individuals and would present with a sore throat and fever. The patient should be questioned about a history of oral sex. The physical exam would reveal a characteristic greenish exudate. Individuals with infectious mononucleosis (B) often present with fever, sore throat, and malaise. A physical exam may demonstrate exudative pharyngeal erythema and hepatosplenomegaly (which often manifests as a left upper quadrant pain). The presence of posterior cervical lymphadenopathy is highly suggestive of infectious mononucleosis, and its absence makes the diagnosis less likely. Viral pharyngitis (D), the most common cause of sore throat, is an inflammation of the pharynx and surrounding tissues with a viral etiology (rhinovirus, adenovirus, coronavirus, echovirus, parainfluenza). Patients would present with coryza, malaise, conjunctivitis, cough, fatigue, and low-grade fever. A rapid streptococcal antigen and throat culture would be negative.

A 23-year-old woman with a history of nasal polyps presents with facial pain, headache, and fever. Her symptoms started approximately 12 days ago with cough, rhinorrhea, and frontal sinus pain, which improved on day 5 but then worsened after day 8. On exam, tenderness to palpation over the frontal sinus is noted as well as purulent nasal drainage. Which of the following organisms is the most common bacterial cause of the suspected diagnosis? A. Moraxella catarrhalis B. Staphylococcus aureus C. Streptococcus pneumoniae D. Streptococcus pyogenes

C. Streptococcus pneumoniae Acute Sinusitis Sx: nasal congestion, pain or pressure over sinuses, ear pain or pressure, headache, fever PE: purulent rhinorrhea Most commonly caused by viral URI If viral, tx: supportive care Bacterial sinusitis: purulent nasal secretions and severe symptoms for ≥ 10 days Amoxicillin or Amoxicillin-clavulanate Moraxella catarrhalis (A) and Streptococcus pyogenes (D) can cause bacterial sinusitis, but they are not the most common bacterial etiologies. In chronic sinusitis with symptoms lasting > 12 weeks, the most common bacterial cause is Staphylococcus aureus (B).

A 30-year-old woman presents to her primary care provider with complaints of widespread, achy musculoskeletal pain, fatigue, and chronic headache for the past 9 months that have severely interfered with her activities of daily living. She states her pain is worse in the morning and aggravated by minor activities. She reports no known sick contacts but states she feels like she has had the flu for 9 months. Her past medical history is significant for anxiety and depression. Physical examination is significant for muscle tenderness over the neck, shoulders, bilateral arms and legs, and upper and lower back. Which of the following is most likely to be found in the patient's history? A. Distal to proximal progression of muscle weakness Insomnia B. Proximal extremity muscle weakness that improves throughout the day C. Recent infection with Campylobacter jejuni D. Insomnia

D. Insomnia Distal to proximal progression of muscle weakness (A) and recent infection with Campylobacter jejuni (D) are both associated with Guillain-Barré syndrome, an autoimmune neuromuscular disorder that results in damage to the myelin sheath of nerve cells. Lambert-Eaton syndrome is a neuromuscular disorder characterized by proximal extremity muscle weakness that improves with use throughout the day (C). It is associated with malignancies, such as small cell lung cancer. Although patients with fibromyalgia may complain of subjective weakness, neurological exam and electrophysiologic testing are usually normal.

A 58-year-old man presents to your office for his annual exam and wants to make sure he is up to date on all of his preventive screenings. His colonoscopy last year was normal. He tells you he started smoking cigarettes at 20 years old and smokes one pack per day. Which of the following is recommended screening for this patient? A. Abdominal ultrasound B. Bone density scanning C. Fecal immunochemical test D. Low-dose computed tomography

D. Low-dose computed tomography The United States Preventive Services Task Force (USPSTF) recommends abdominal ultrasound (A) to screen for abdominal aortic aneurysm in men between the ages of 65 and 75 years when they have any smoking history. This is a one-time test unless there are findings of aneurysm. The USPSTF recommends bone density scanning (B) to screen for osteoporosis in women at average risk starting at age 65 years. Earlier screening starting at age 60 years may be indicated in women with additional risk factors for osteoporosis. Fecal immunochemical test (C) is one of the tools used to screen for colorectal cancer in patients aged 50-75 years as recommended by the USPSTF. For those patients who do not want to undergo colonoscopy and who are of average risk, it is a recommended option and is done annually. Those who have already been screened with colonoscopy should follow the repeat recommendations and do not need to be additionally screened with the fecal immunochemical test unless they are outside the window of the repeat recommendations for their next colonoscopy.

A 38-year-old woman presents to her primary care provider with complaints of recurrent episodes of vertigo and tinnitus for the past 4 months. She states this has never happened before. Her episodes last about 2 hours and are accompanied by nausea. She is put on a low-sodium diet and prescribed triamterene-hydrochlorothiazide, which significantly improves her symptoms. Which of the following additional physical examination findings is most consistent with the suspected diagnosis? A. High-frequency conductive hearing loss B. High-frequency sensorineural hearing loss C. Low-frequency conductive hearing loss D. Low-frequency sensorineural hearing loss

D. Low-frequency sensorineural hearing loss Ménière Disease Patient presents with episodic low-frequency hearing loss, tinnitus with aural (ear) fullness, and vertigo lasting 20 minutes up to 24 hours Diagnosis is made clinically Most commonly caused by too much inner ear endolymph and increased pressure within the inner ear Treatment is low-salt diet, diuretics (HCTZ + triamterene) High-frequency (A) and low-frequency conductive hearing loss (C) are incorrect because Ménière disease is associated with sensorineural hearing loss. Causes of conductive hearing loss include cerumen impaction, otitis media, otitis media with effusion, and cholesteatoma. Additionally, low-frequency hearing loss is much more common than high-frequency conductive hearing loss. Presbycusis typically presents with high-frequency sensorineural hearing loss (B) before affecting lower frequencies. Ménière disease presents with low-frequency sensorineural hearing loss that will progress to affect all frequencies in 8-10 years.

A 55-year-old man presents with complaints of daytime sleepiness. The patient denies insomnia or nighttime awakenings. The patient does report that his wife complains about his loud snoring, but that has been going on for years. Which of the following findings would support your suspected diagnosis? A. Body mass index 25 kg/m2 B. BP 120/80 mm Hg C. Hemoglobin of 11 g/dL D. Neck circumference of 18 inches

D. Neck circumference of 18 inches A body mass index (BMI) of 25 kg/m2 (A) is not a risk factor for obstructive sleep apnea. Obesity with a BMI over 30 kg/m2 is a risk factor. A blood pressure of 120/80 mm Hg (B) is normal and is not associated with obstructive sleep apnea. Patients with uncontrolled hypertension should be considered for an evaluation of sleep apnea if the cause of the uncontrolled hypertension is unknown. A hemoglobin of 11 g/dL (C) is below normal levels and is not a risk factor. Patients with an elevated hemoglobin and hematocrit of unknown cause should be evaluated for obstructive sleep apnea. Obstructive Sleep Apnea Risk factors: obesity, male gender, advancing age, allergies Sx: apneic episodes while sleeping, snoring, and daytime sleepiness PE: enlarged tonsils, crowded oropharynx, large neck circumference, hypertension Diagnosis is made by sleep study Treatment is weight loss, CPAP First-line treatment in children is adenotonsillectomy Adverse outcomes: CVA, motor vehicle accidents, pulmonary hypertension, cor pulmonale (right ventricular hypertrophy)

A 25-year-old woman presents with complaints of intermittent headache. The patient describes the headache as a band tightening around her head. Headaches are relieved with ibuprofen. Which of the following additional physical exam findings would be present given the most likely diagnosis? A. Headache aggravated by routine physical activity B. Nasal congestion C. Nausea and vomiting D. Pericranial muscle tenderness

D. Pericranial muscle tenderness Tension Headache Patient presents with bilateral, nonpulsating, band-like pain PE will show neck muscle tenderness Most commonly caused by stress Treatment is NSAIDs (abortive), TCAs (preventive) Most common type of headache Headaches that are aggravated by routine physical activity (A) are characteristic of migraines and not tension headaches. Tension headaches are aggravated by stress, fatigue, glare, and noise. Nasal congestion (B) can be associated with sinus headaches or with cluster headaches if it occurs on the ipsilateral side of the cluster pain. Nausea and vomiting (C) is associated with migraine headaches as opposed to tension-type headaches. Patients with migraines typically present with a unilateral pulsating headache of moderate to severe intensity that is aggravated by routine physical activity. Patients with migraine headaches present with nausea or vomiting and photophobia or phonophobia.

A 46-year-old man presents to his primary care provider for a follow-up on his laboratory results. His fasting glucose level is 232 mg/dL, and his hemoglobin A1C is 9.2%. His past medical history is significant for a treated hepatitis B infection and diabetes mellitus type 2. Which of the following should be recommended to the patient at this time? A. Hepatitis B vaccination B. Monthly measurement of A1C level C. Ophthalmology follow-up in 5 years D. Pneumococcal vaccine

D. Pneumococcal vaccine Adult Pneumococcal Vaccines Indicated in adults ≥ 65 years or < 65 years with risk factors For those who have not previously received PCV (or vaccine status unknown): One dose of PCV20 or PCV15 should be given If PCV 15 is given, administer PPSV23 at least 1 year later (except in patients with certain risk factors where an interval ≥ 8 weeks can be considered) For those who previously received PCV13 but not PPSV23: Administer PPSV23 at least 1 year after PCV13 Vaccines are inactivated The hepatitis B vaccination (A) is recommended for unvaccinated adults < 60 years of age who have not been previously infected with hepatitis B. Monthly measurement of A1C level (B) is incorrect. The hemoglobin A1C is a measure of the amount of glucose a red blood cell is exposed to over its lifespan. It should not be measured more frequently than every 3 months, which is the lifespan of the red blood cell. A1C levels should be measured every 3 months until the patient is at goal. When a patient is at goal, the A1C can be measured every 6 months. An ophthalmology follow-up in 5 years (C) is the appropriate recommendation for a patient diagnosed with type 1 diabetes mellitus. Patients with type 2 diabetes should have yearly follow-up if there is retinopathy and every 2 years if there is not.

A 16-year-old girl presents with reports of pain with menstrual cycles for the past 3 months. The pain is described as a cramping in her low abdomen that begins at the onset of menstruation and lasts for the first 2 to 3 days of her menstrual cycle. The pain is dull and intermittent with radiation to her low back at times. She reports that she has never been sexually active. Menarche was at age 11. Which of the following is the most likely diagnosis? A. Endometriosis B. Leiomyoma C. Pelvic inflammatory disease D. Primary dysmenorrhea

D. Primary dysmenorrhea Endometriosis (A) is a condition that occurs when normal endometrial tissue migrates outside of the uterine cavity and causes a triad of symptoms, including dysmenorrhea, dyschezia, and dyspareunia. Endometriosis is associated with pain that worsens over time and is most commonly cyclic in nature although the pain can be noncyclic, occurring at times other than menstruation. This is a cause of secondary dysmenorrhea and typically affects women over the age of 20. Leiomyoma (B), or uterine fibroids, usually do not cause pain or cramping. These are associated with heavy menstrual bleeding and are more common in older women. Pelvic inflammatory disease (C) is commonly caused by a sexually transmitted infection and patients will report purulent vaginal discharge and pelvic pain. This is an acute condition that can be associated with signs of systemic infection, such as fever.

A 32-year-old woman is hospitalized for bilateral pulmonary embolisms following a period of immobilization after suffering a femur fracture. She has a family history of blood clots. Her medical team makes the decision to start her on warfarin while first bridging her with heparin. Several days into her admission, the heparin is discontinued. The patient develops large, demarcated violet areas of necrotic skin tissue on her breasts. What hypercoagulable disorder should be considered given the course of the patient's illness? A. Antiphospholipid syndrome B. Antithrombin III deficiency C. Factor V Leiden D. Protein C deficiency

D. Protein C deficiency While antiphospholipid syndrome (A) is a hypercoagulable disorder, it is not inherited but rather an autoimmune syndrome. Patients may also have other autoimmune conditions, such as systemic lupus erythematosus. Antithrombin III deficiency (B) is also a hypercoagulable disorder, but it is not as frequently associated with warfarin-induced skin necrosis as protein C deficiency. Additionally, antithrombin III deficiency is sometimes associated with heparin resistance. While factor V Leiden (C) is the most common hypercoagulable disorder, it is not as commonly associated with warfarin-induced skin necrosis as protein C deficiency. Warfarin-induced skin necrosis is a complication of warfarin therapy that can be observed in patients with protein C deficiency. Protein C deficiency is a hypercoagulable disorder that can be inherited or acquired. Patients with protein C deficiency have an increased risk of venous thromboembolism (VTE), particularly deep vein thrombosis in the leg, pulmonary embolism, and thrombosis within the mesenteric veins. The risk of VTE increases with prolonged immobilization, surgery, smoking, pregnancy, and use of oral contraceptives or exogenous estrogen. In addition to VTE, adverse pregnancy outcomes, such as fetal loss and miscarriage, can be observed. Patients with protein C deficiency can also develop warfarin-induced skin necrosis due to the transient hypercoagulable state secondary to depletion of protein C that occurs when first starting warfarin. Neonatal purpura fulminans can also be observed. The diagnosis of protein C deficiency is often suspected in patients with recurrent VTE, VTE in an unusual location, VTE occurring in a patient younger than 50 years of age, patients with a significant family history of VTE, or patients with warfarin-induced skin necrosis. Low protein C levels (< 4 mcg/mL) are observed but can be normal in some patients. The level should be obtained after recovery from an inflammatory event or illness. Additionally, anticoagulants such as warfarin can interfere with protein C levels. Functional assays are used to measure the protein C activity in the plasma. Anticoagulation should be started in patients with protein C deficiency who experience a thromboembolic event. Direct oral anticoagulants are more commonly used in patients with typical VTE presentations, while warfarin may be preferred for patients with unusual or severe presentations or significant clot size. The duration of anticoagulation depends on whether or not the thromboembolism was provoked or unprovoked. Patients with an unprovoked thromboembolism with an identified thrombophilia, such as protein C deficiency, may be started on anticoagulation indefinitely. For patients with warfarin-induced skin necrosis, warfarin should be discontinued immediately. The warfarin antagonist, vitamin K, should be administered, and unfractionated heparin can be given to continue anticoagulation. Additionally, protein C should be administered to the patient either via protein C concentrate or through fresh frozen plasma.

A 25-year-old woman presents after experiencing a depressed mood, loss of pleasure in almost all activities, weight loss, poor concentration, and insomnia for the last 2 weeks. Prior to this, she had a period of euphoric mood for 10 days with associated inflated sense of self, decreased need for sleep, and racing thoughts. She was unable to work because of the severity of these symptoms. She reports no substance use, and her medical history is benign. Which of the following would be the most appropriate FDA-approved treatment for this patient at this time? A. Carbamazepine B. Lamotrigine C. Lithium D. Quetiapine

D. Quetiapine Carbamazepine (A) is an older mood stabilizer that is not generally a first-line treatment of choice due to its many drug-drug interactions, and it is not FDA-approved for bipolar depression. Lamotrigine (B) is indicated for maintenance treatment for bipolar disorder but is not FDA approved for acute bipolar depression, and there is a risk of Stevens-Johnson syndrome with this medication. Lithium (C) is approved for the treatment of acute mania, mixed mood episodes, and maintenance treatment in bipolar disorder, but it is not FDA approved for bipolar depression. It still may be recommended in combination with other medications, as noted above.

A 19-year-old man presents to his primary care provider with recurrent episodes of mild intermittent jaundice. He states these episodes began several years ago and seem to recur when he is stressed. He reports no other symptoms. Physical examination reveals mild scleral icterus and yellowing of the skin. Laboratory testing is significant for a slightly elevated indirect bilirubin. Which of the following best describes the pathophysiology of the most likely diagnosis? A. Cholestasis B. Reduced excretory function of hepatocytes C. Reduced hepatic reuptake of bilirubin conjugates D. Reduced production of uridine 5'-diphospho-glucuronosyltransferase

D. Reduced production of uridine 5'-diphospho-glucuronosyltransferase Cholestasis (A) is a condition in which there is retention of bile in the liver. Conjugated hyperbilirubinemia can occur from cholestasis. Intrahepatic cholestasis of pregnancy is usually seen in the third trimester. Reduced excretory function of hepatocytes (B) describes the pathophysiology of Dubin-Johnson syndrome. Dubin-Johnson syndrome is a benign, autosomal recessive condition characterized by conjugated hyperbilirubinemia. Characteristic findings associated with Dubin-Johnson syndrome are an inability to visualize the gallbladder on oral cholecystography and the presence of a black liver caused by lack of multidrug resistance-associated protein 2. Reduced hepatic reuptake of bilirubin conjugates (C) describes the pathophysiology of Rotor syndrome. Rotor syndrome is a benign, autosomal recessive condition with conjugated hyperbilirubinemia. It is similar to Dubin-Johnson syndrome except the gallbladder can be visualized on oral cholecystography and there is no black liver.

A 40-year-old woman presents with fatigue. She notes it takes her an hour to fall asleep at night, and she wakes up frequently during the night. This has been ongoing for the past year and began when she started a new job. She thinks it has gotten worse over time, which has prompted her visit. Which of the following would be the best next step in managing this patient's care? A. Order polysomnography B. Prescribe oral temazepam C. Recommend a bedtime antihistamine D. Refer for cognitive behavioral therapy

D. Refer for cognitive behavioral therapy Ordering polysomnography (A) would be appropriate in cases where an organic cause of insomnia is suspected, such as obstructive sleep apnea. Prescribing oral temazepam (B) should not be a first-line approach, as this is not as effective as cognitive behavioral therapy and includes side effects such as impaired motor coordination and alertness. Recommending a bedtime antihistamine (C) may be helpful in initiating sleep, but patients can develop a tolerance over time to these drugs, and they have unwanted anticholinergic side effects.

A 50-year-old man presents to the office for evaluation of progressive dyspnea on exertion and a persistent nonproductive cough. Symptoms began about 6 months ago. He has no significant medical history other than atrial fibrillation, which has been well-controlled on amiodarone for the past 12 months. He does not take any other medications. He reports no history of tobacco or illicit drug use or exposure to occupational agents. On physical examination, his heart rate and rhythm are regular, and no murmur or gallop is heard. Inspiratory crackles are noted diffusely in both lungs. His jugular venous pressure is normal, and he has no peripheral edema. Which of the following findings is most likely to be seen on chest radiography? A. Bilateral hilar adenopathy B. Kerley B-lines C. Pleural plaques D. Reticular opacities

D. Reticular opacities Bilateral hilar adenopathy (A) is a common radiographic finding seen in pulmonary sarcoidosis. Kerley B-lines (B) are linear opacities that are about 1 cm in length and 1 mm in thickness with a predilection for the lower lung zones abutting a pleural surface. They result from thickening of the subpleural interstitium. They are seen in chronic left ventricular failure, mitral valve disease, lymphangitic carcinomatosis, asbestosis, and some viral pneumonias. Pleural plaques (C) are deposits of hyalinized collagen fibers that are indicative of asbestos exposure and thus a radiographic finding seen in asbestos-related pulmonary disease. This patient has interstitial lung disease. Interstitial lung disease is a term used to refer to the diffuse parenchymal lung diseases that are classified together because of similar clinical, radiographic, physiologic, or pathologic manifestations. Diffuse parenchymal lung diseases consist of disorders of known cause (such as rheumatic disease, environmental, or drug-related) and disorders of unknown cause (such as idiopathic interstitial pneumonias and granulomatous lung disorders). The most common identifiable causes of interstitial lung disease are exposure to occupational and environmental agents, drug-induced pulmonary toxicity, and radiation-induced lung injury. One medication that is notorious for causing interstitial lung disease is the antidysrhythmic agent amiodarone. The onset of symptoms is usually within 6 to 12 months of starting amiodarone but may occur within 2 months to several years of starting the medication. Patients with interstitial lung disease often report progressive dyspnea on exertion and a persistent nonproductive cough. Lung examination often reveals crackles. Clubbing of the digits may also be noted on physical examination. Patients with interstitial lung disease display lung function abnormalities on spirometry, particularly a restrictive ventilatory pattern (reduced forced vital capacity and total lung capacity with a normal or increased FEV1/FVC ratio). Chest imaging is abnormal in these patients as well. Chest X-ray may be useful in suggesting the presence of interstitial lung disease and can be useful to rule out other differential diagnoses. The most common radiographic abnormality seen on chest X-ray among patients with interstitial lung disease is a reticular pattern. Nodular or mixed patterns may be seen as well. High-resolution computed tomography is the best diagnostic test to confirm interstitial lung disease. Certain high-resolution computed tomography findings help to narrow the differential diagnosis of interstitial lung disease. Airspace opacities, ground glass opacities, honeycombing, reticular opacities, or reticulonodular opacities may be noted depending on the type of interstitial lung disease present.

A 52-year-old man presents to urgent care with complaints of fever, urinary retention, and irritation with voiding. He reports he was hospitalized 1 week ago and had a urinary catheter in for 5 days that was just removed yesterday. Vital signs are T 101.6°F, BP 122/74 mm Hg, HR 89 bpm, RR 18 breaths per minute, and oxygen saturation 100% on room air. His urinalysis is significant for positive nitrite, 3+ leukocyte esterase, 10 WBC/hpf, 10 RBC/hpf, and 2+ bacteria. Which of the following findings on physical exam would suggest a diagnosis of prostatitis? A. Hard, nodular, asymmetrical prostate B. Uniform, enlarged, firm prostate with a palpable median sulcus C. Unilateral costovertebral angle tenderness D. Warm, edematous, exquisitely tender prostate

D. Warm, edematous, exquisitely tender prostate Prostatitis is an inflammatory condition most commonly caused by migration of Escherichia coli up the urethra to the prostate. Other bacterial organisms that may cause prostatitis include Pseudomonas, Proteus, and members of the Enterobacteriaceae family (e.g., Klebsiella, Enterobacter, Serratia). Risk factors include urinary catheterizations, urogenital instrumentation, anatomical abnormalities (e.g., urethral stricture), and prostate biopsy. Clinical presentation includes fever, irritative voiding symptoms, and perineal, sacral, or suprapubic pain. Urinary retention may also be present. A rectal examination will reveal a warm, edematous, exquisitely tender prostate. Clinicians should avoid rigorous manipulation of the prostate, which increases the risk of septicemia. Laboratory findings include leukocytosis, left shift, pyuria, bacteriuria, and hematuria. A urine culture is indicated for identification of the offending bacterium and antibiotic guidance. Blood cultures may be collected if signs of sepsis are present. Any indwelling catheters should be removed. Empiric antibiotic therapy for patients in whom hospitalization (e.g., existing comorbidities, symptoms of sepsis) is warranted includes intravenous administration of ampicillin and an aminoglycoside (e.g., gentamicin). Patients who are afebrile for 48 hours may be transitioned to a 6-week course of oral antibiotics (e.g., trimethoprim-sulfamethoxazole, ciprofloxacin). In-and-out catheterization or a short-term indwelling urinary catheter may be used to alleviate urinary retention. Patients without major comorbidities or symptoms of sepsis may be managed with oral antibiotics in the outpatient setting. A hard, nodular, asymmetrical prostate (A) on digital rectal exam is concerning for prostate cancer, especially if the prostate-specific antigen level is > 10 ng/mL. A uniform, enlarged, firm prostate with a palpable median sulcus (B) on digital rectal exam is associated with benign prostatic hyperplasia. Symptoms associated with benign prostatic hyperplasia include slow urinary stream, straining to void, urinary intermittency, and terminal dribbling. Unilateral costovertebral angle tenderness (C) is associated with pyelonephritis. Patients with prostatitis tend to present with perineal, sacral, or suprapubic pain rather than costovertebral angle tenderness.

A 70-year-old man with a 50 pack-year history of cigarette smoking is being evaluated for possible community-acquired pneumonia. He has been coughing for 2 days but is currently afebrile. Physical exam is unremarkable. His chest radiograph reveals a single, well-circumscribed nodule, approximately 1 cm in diameter, at the periphery of the right lung. Which of the following is the best next step in management? A. Computed tomography scan B. Positron emission tomography scan C. Repeat chest imaging in 3 months D. Review of a previous chest radiograph

D. Review of a previous chest radiograph A computed tomography scan (A) is indicated if there are no previous films available for comparison, if the nodule has increased in size for more than 2 years, if there are findings suggestive of cancer or cancer-related complications on a chest radiograph, or if there is no nodule on the previous chest radiograph. Nodules that are 1 cm or larger require evaluation using a positron emission tomography scan (B). If the scan is positive, a biopsy of the nodule should be performed to evaluate for malignancy. Repeat chest imaging in 3 months (C) is reserved for nodules that are found to be benign based on CT scan findings. A pulmonary nodule is a single, well-circumscribed nodule that is usually discovered incidentally, often with no mediastinal or hilar lymph node involvement. It is important to distinguish between malignant and benign nodules. A malignant nodule is most likely to be present in an older individual (> 60 years of age) and is associated with a history of smoking. It is more likely to be larger in size (> 2 cm) and have irregular borders with eccentric asymmetrical calcification. An enlarging nodule suggests malignancy. Once a pulmonary nodule is discovered radiographically, a review of a previous chest X-ray should be done for comparison. If the lesion is stable for more than 2 years, it is likely benign. A nodule showing a sizable increase over a period of days is usually nonmalignant (likely infectious or inflammatory). Whereas malignant lesions tend to grow rapidly over a period of months. The following features should prompt further evaluation for lung cancer regardless of symptoms: chest radiograph demonstrating a new or enlarging focal lesion, a pleural effusion, pleural nodularity, enlarged hilar or paratracheal nodes, endobronchial lesion, postobstructive pneumonia, or segmental or lobar atelectasis. Nodules suspected to be malignant require evaluation using chest CT scan, followed by biopsy or early resection.

A 19-year-old woman presents to her primary care provider with concerns about a lesion on her finger for 2 months. She reports no symptoms other than mild tenderness with pressure over the lesion. She states she is very self-conscious about the lesion and would like to get rid of it as soon as possible. Physical examination reveals the presence of the lesion shown in the image above. Which of the following is the most appropriate initial clinical intervention? A. Injection of Clostridium toxin B. Injection of diluted bleomycin C. Occlusion of the lesion with duct tape D. Salicylic acid plaster after paring

D. Salicylic acid plaster after paring Injection of Clostridium toxin (A), also known as botulinum toxin, is most commonly used for reducing skin wrinkles. It is not indicated in the treatment of cutaneous warts. Botulinum toxin is also used to treat cervical dystonia, hyperhidrosis, overactive bladder, and amblyopia. Injection of diluted bleomycin injection (B) is not recommended for warts on the fingers because of increased risk of Raynaud phenomenon, nail loss, and terminal digital necrosis. Occlusion of the lesion with duct tape (C) alone is less effective than cryotherapy or salicylic acid plaster.

An 18-year-old woman presents with her parents who are concerned about her dietary habits. She has been binge eating and then either vomiting or using laxatives as a compensatory behavior for the past several months. Which of the following historical findings would be consistent with the most likely diagnosis? A. Attention-deficit disorder B. Narcissistic personality disorder C. Schizoid personality disorder D. Specific phobia disorder

D. Specific phobia disorder Bulimia Nervosa DSM-5: recurrent episodes of binge eating followed by inappropriate compensatory behavior via self-induced vomiting, laxative misuse, excessive exercise, or caloric restriction (occurring at least once per week for 3 months) Sense of lack of control during eating episodes Self-evaluation is unduly influenced by body shape or weight PE: body weight usually within or above normal range, dental erosions, parotid gland swelling, callused knuckles Tx options: cognitive behavioral therapy, fluoxetine or other SSRIs, or combined CBT/pharmacotherapy Attention-deficit disorder (A) is not associated with the development of bulimia nervosa, although impulsivity is a common personality trait found in both bulimia nervosa and attention-deficit disorder. Narcissistic personality disorder (B) is characterized by an overexaggerated sense of self-importance and a lack of empathy for others. Bulimia nervosa is associated with the need for attention and admiration but lacks the other characteristics of narcissistic personality disorder. Schizoid personality disorder (C) is associated with detachment from social relationships and limited ability to express emotions. Social anxiety disorder is associated with bulimia nervosa but differs from schizoid personality disorder in that individuals with social anxiety disorder fear rejection and criticism in social situations.

A 35-year-old man presents to the urgent care with 2 weeks of a nonproductive cough. He initially had rhinorrhea, sore throat, and malaise, but these symptoms have since resolved. He reports no history of smoking cigarettes or prior chronic obstructive lung disease diagnosis. Vital signs are T 99.2°F, HR 84 bpm, BP 120/80 mm Hg, RR 20 breaths per minute, and oxygen saturation 98% on room air. Physical exam reveals clear breath sounds. Chest X-ray shows no acute infiltrate. Which of the following is the recommended intervention? A. Amoxicillin and azithromycin B. Doxycycline C. Prednisone D. Supportive care

D. Supportive care Acute Bronchitis Patient presents with a productive cough for > 5 days Most commonly caused by viruses Treatment is symptomatic management Most common cause of minor hemoptysis Routine Abx therapy not indicated Amoxicillin and azithromycin (A) may be used in combination in the outpatient treatment of community-acquired pneumonia. Amoxicillin provides coverage against typical organisms, such as Streptococcus pneumoniae, while azithromycin provides coverage against atypical organisms, such as Legionella and Mycoplasma pneumoniae. Acute bronchitis does not require antibiotic treatment since it is caused by viruses. Doxycycline (B) provides coverage against typical and atypical organisms in areas where the Streptococcus pneumoniae resistance rate is acceptable. Antibiotics are not recommended for the treatment of acute bronchitis. Prednisone (C) is a corticosteroid. Steroids are not recommended in the treatment of acute bronchitis because studies show no benefit and they have possible adverse effects.

A 23-year-old woman presents to the office with increasing fatigue and inability to concentrate. She has been gaining a lot of weight in her face and abdomen and has had multiple yeast infections over the past year. The patient reports no history of exogenous steroid use. A physical exam reveals some purple striae around her abdomen and thighs. She also has very thin extremities. Initial testing reveals elevated cortisol levels and no suppression with low-dose dexamethasone suppression test. Which of the following further tests will confirm the diagnosis of Cushing disease? A. Low serum adrenocorticotropic hormone levels B. No change after corticotropin-releasing hormone stimulation test C. No suppression with high-dose dexamethasone suppression test D. Suppression with high-dose dexamethasone suppression test

D. Suppression with high-dose dexamethasone suppression test Cushing Syndrome Patient presents with amenorrhea, central obesity, depressive symptoms, and easy bruising PE will show purple striae, moon face (facial adiposity), buffalo hump (increased adipose tissue in the neck and upper back), and hypertension Diagnosis is made by 24-hour urine cortisol and testing ACTH levels Most noniatrogenic cause is hypercortisolism from ACTH-secreting pituitary tumor If cause is pituitary tumor then it's called Cushing disease Low serum ACTH levels (A) is incorrect since serum levels of ACTH are elevated in Cushing disease. Low levels of ACTH are present in the setting of glucocorticoid-secreting adrenal tumors since the HPA axis would be suppressed. No change after CRH stimulation test (B) is incorrect since ACTH and cortisol levels are elevated 45 minutes after administration of CRH in patients with Cushing disease. No suppression with high-dose dexamethasone suppression test (C) is incorrect since the high-dose dexamethasone suppression test does suppress glucocorticoid production in patients with Cushing disease. This is because pituitary adenomas are only partially resistant to negative feedback by glucocorticoids. With ectopic ACTH-secreting tumors, cortisol suppression does not occur with the high-dose dexamethasone suppression test.

A 22-year-old man in the ED waiting room suddenly loses consciousness and falls to the ground. After a period of 30 seconds, he begins to convulse. His convulsions stop spontaneously after 2 minutes. The patient is noted to be confused and disoriented. He has urinated on himself and has a bloody lip. Which of the following is the most likely diagnosis? A. Absence seizure B. Epileptic spasm C. Status epilepticus D. Tonic-clonic seizure

D. Tonic-clonic seizure An absence seizure (A) typically occurs in children between 5-10 years of age and is associated with brief episodes of staring into space. An epileptic spasm (B) usually occurs in infancy and presents with sudden flexion or extension of the truncal muscles. Status epilepticus (C) is characterized by a single seizure of > 5 minutes' duration or two or more seizures within a 5-minute period without recovery of consciousness between seizures.

A 35-year-old woman presents with increased malodorous vaginal discharge for 2 days. She has multiple sexual partners and does not practice safe sex. On exam, a frothy, greenish-yellow vaginal discharge is noted that has a pH of 6.0. Hyperemic, friable cervical mucosa is also noted on a speculum exam. Which of the following organisms is most likely to be present? A. Candida albicans B. Gardnerella vaginalis C. Neisseria gonorrhoeae D. Trichomonas vaginalis

D. Trichomonas vaginalis Candida albicans (A) causes vulvovaginal candidiasis (yeast infection). Patients typically present with vulvar or vaginal itching and burning with external dysuria. Odorless thick, cottage cheese-like discharge may also be present. A wet mount of the discharge would demonstrate budding yeast, and the pH would be acidic. Bacterial vaginosis is most commonly caused by Gardnerella vaginalis (B). Patients usually present with increased vaginal discharge that often has a noticeable fishy odor. Physical exam reveals a thin, ivory to gray homogeneous discharge with a pH > 4.5. Clue cells would be present on a wet mount. Neisseria gonorrhoeae (C) causes a sexually transmitted infection affecting the mucous membranes of the urethra, cervix, uterus, and fallopian tubes in women. Patients would present with a white, yellow, or greenish urethral discharge, dysuria, or may be asymptomatic. Microscopy would reveal nonmotile, gram-negative diplococci in pairs.

A 40-year-old man presents to his primary care provider for an annual physical. He has no significant past medical history and has no concerns today. His blood pressure is 140/80 mm Hg, and all other vitals are within normal limits. Physical exam reveals a man who is obese with a waist circumference of 115 cm. What other value would qualify this patient for metabolic syndrome? A. Body mass index of 30 kg/m2 B. Fasting blood glucose of 95 mg/dL C. Low-density lipoprotein of 150 mg/dL D. Triglyceride of 180 mg/dL

D. Triglyceride of 180 mg/dL A BMI score of 30 kg/m2 (A) is considered to be obese, however, BMI scores are not part of the diagnostic criteria for metabolic syndrome. A fasting blood glucose of 95 mg/dL (B) is considered to be within normal limits. An impaired fasting blood glucose is a value greater than 100 mg/dL, which is a criterion for metabolic syndrome. A low-density lipoprotein (C) value greater than 100 mg/dL is considered high, however, abnormal LDL values are not one of the criteria for metabolic syndrome.

A 21-year-old woman presents to an obstetrician after a positive home pregnancy test. She states she has a history of irregular menses and her last menstrual period was 6 weeks ago. Her beta-human chorionic gonadotropin is 46,000 mIU/mL. A transvaginal ultrasound is shown above and estimates a 7 week gestational age. Which of the following tests should always be ordered at this time? A. Alpha-fetoprotein B. Estradiol C. Inhibin A D. Urinalysis

D. Urinalysis Alpha-fetoprotein (A), estradiol (B), inhibin A (C), and human chorionic gonadotropin are tests that comprise the maternal quad screen, which may be performed at 16-20 weeks gestation.

A 21-year-old man presents to the emergency department via ambulance after his mother found him seizing early this morning. The patient's mother reports she heard a choking sound coming from her son's room around 4:00 AM. She reports his arms and legs were jerking for at least 1 minute, and he was not responsive. He urinated on himself. In the emergency department, he appears to be postictal, and the oropharynx examination reveals a tongue laceration. Toxicology screen is negative, and finger stick blood glucose is within normal range. Upon further questioning, the patient's mother reports he had a similar episode 2 years ago, but it was determined he did not need any further intervention at that time. Which of the following is the preferred pharmacologic intervention, given the most likely diagnosis? A. Carbamazepine B. Phenytoin C. Topiramate D. Valproate

D. Valproate Carbamazepine (A) is preferred pharmacologic therapy for focal onset seizures and can worsen generalized epilepsy. While phenytoin (B) is often used in status epilepticus and for generalized tonic-clonic seizures, it can worsen primary generalized epilepsy and other generalized epilepsy syndromes. While topiramate (C) is appropriate for generalized seizures, it is not first line for generalized epilepsy.

A 52-year-old woman with a past medical history of Raynaud phenomenon presents to the clinic with sudden-onset chest pain, shortness of breath, and anxiety that began 15 minutes ago while she was sleeping and lasted approximately 3 minutes. At the time of this visit, the pain has resolved. Current vital signs are BP 122/80 mm Hg, HR 118 beats per minute, RR 28 breaths per minute, and oxygen saturation 96% on room air. ECG is unremarkable, and serial troponin testing is negative. Which of the following is the most likely diagnosis? A. Aortic dissection B. Non-ST elevation myocardial infarction C. ST elevation myocardial infarction D. Vasospastic angina

D. Vasospastic angina Aortic dissection (A) can cause chest pain that is typically described as "tearing," with radiation to the back and unequal pulses in the upper extremities. The pain of aortic dissection is unlikely to spontaneously resolve, as occurred in the patient described in the vignette above. Non-ST elevation myocardial infarction (B) would also not show ST elevations on ECG, but serial troponins would be positive. ST elevation myocardial infarction (C) would show ST elevations in several leads with reciprocal changes, and serial troponins would be positive.


संबंधित स्टडी सेट्स

Completing the Application, Underwriting, and Delivering The Policy

View Set

AP Human Geography Chapter 2 Test: Population and Health

View Set

CISA PT 4: INFO SYS OPERATIONS AND BUSINESS RESILIENCE

View Set

Accounting 201 Chapter 10 Wiley Quiz

View Set

TLB-Chapter 39: Oxygenation and Perfusion

View Set

Final Exam - Mgmt Theory & Practice

View Set

EXAM #2 CENGAGE BRIEF HYPOTHETICALS (CHP 11-17)

View Set